Está en la página 1de 130

L&T Q & A-I

WRITTEN TEST
Total 90 questions , 90 minutes ---SECTIONAL CUTOFF IS THERE
1. Aptitude (30 quest)
2. Reasoning (30 quest)
3. English (30 quest)
Aptitude (30 quest)
1. From a vessel, 1/3rd of the liquid evaporates on the first day. On the second day 3/4th of
the remaining liquid
evaporates. What fraction of the volume is present at the end of the second day.
Ans: 50%
2. An orange glass has orange juice and white glass has apple juice both of equal volumes.
50 ml of the orange juice is
taken and poured into the apple juice. 50 ml from the white glass is poured into the
orange glass. Of the two
quantities, the amount of apple juice in the orange glass and the amount of orange juice
in the white glass, which one
is greater and by how much?
Ans: The two quantities are equal
3. There is a 4 inch cube painted on all sides. This is cut down into of 1 inch cubes. What is
the no of cubes which
have no pointed sides?
Ans: 8
4. Sam and Mala have a conversation. Sam says I am certainly not over 40. Mala says I am
38 and you are
at least 5 years older than me. Now, Sam says you are atleast 39. All the statements by
the two are false.
How old are they really?
Ans: Mala = 38 yrs; Sam = 4 yrs
5. There is a certain four digit number whose fourth digit is twice the first digit. Third digit is
three more than
second digit. Sum of the first and fourth digits twice the third number. What was that
number?
Ans: 2034 and 4368
. In a railway station, there are two trains going. One in the harbour line and one in the
main line, each having a
frequency of 10 minutes. The main line service starts at 5 o'clock and the harbour line
starts at 5.02A.M. A man
goes to the station every day to catch the first train that comes. What is the probability of
the man catching the
first train?
Ans: 08
7. A family X went for a vacation. Unfortunately it rained for 13 days when they were there.
But whenever it rained
in the mornings, they had clear afternoons and vice versa. In all they enjoyed 11
mornings and 12 afternoons.
How many days did they stay there totally?
Ans: 8
8. There is a safe with a 5 digit No. The 4th digit is 4 greater thansecond digit, while 3rd
digit is 3 less than 2nd digit.
The 1st digit is thrice the last digit. There are 3 pairs whose sum is 11. Find the number.
Ans: 65292
9 Bird is flying 120 km/hr between B to R. Two trains at B to R at 0 kmph. The distance
traveled by the bird
before it is killed?
Ans: 20
10. Four persons have to cross the bridge they are having one torch light. Four persons take
1,2,5,10 minutes
respectively, when two persons are going the will take the time of the slowest person
what is the time taken to
cross all of them.
Ans: minutes
11 A vender solds two things at same cost 12 Rs with one item at 25% profit and other at 20% loss,
by this
transaction he made profit or loss by how much?
Ans: Loss,60%
12. Two friends A and B are running up hill and then to get down. Length of road is 440
yards. A on his return journey
met B going up at 20 yards from top. If A has finished the race 5 minutes earlier than
B, then how much time A
had taken to complete the race?.
Ans: 63 minutes
13. In the month of October in a year has exactly four Mondays and four Fridays, find what
day of week will be on the
20th of November of that year.
Ans: 20th November was a wednesday
14.Six persons A,B,C,D,E & F went to solider cinema. There are six consecutive seats. A sits
in one of the seats
followed by B, followed by C and soon. If a taken one of the six seats , then B should sit
adjacent to A. C
should sit adjacent A or B. D should sit adjacent to A, B,or C and soon. How many
possibilities are there?
Ans: 32 ways
15 4 mathematician has x apples. If he arranges them in rows of 3 one will be left. The
same is the case with
5,7,9 apples. But when he arranged them in rows of 11, non will be left. Find the no. of
apples.
Ans: 946
1 A merchant in the last day sells 2 lamps for Rs.12 price. He finds that he has got 25 %
gain on one and 20%
lost on the other. Did he loose or gain overall? If so how much?
Ans: 60 paise Loss
17. Mr. X got Paramvir Chakra in 1971. He died when his age was 1/59th the year of birth.
What was his date of
birth?
Ans: 94
18. There are 51 number of balls in a bag. Out of which one is heavy ball. In how many
minimum number of
weighing you can find the heavy ball.
Ans: 8
19. The profit made by a company in one year is enough to give % return on all shares.
But as the preferred
shares get on return of 7.5%, so the ordinary shares got on return of 5%. If the value of
preferd shares is
Rs 4,000000, then what is the value of ordinary shares?
Ans: Rs 6,000000
20. A & B two places. C & D are two people. C started from A and D started from B. When
they meet each
other in the way C traveled 18 m more than D. Then C takes 13 and half a minute and
D takes 24 minutes to
reach the other end. What was the distance between A & B.
Ans: 26(
21. A family I know has several children. Each boy in this family has as many sisters as
brothers but each girl has
twice as many brothers as sisters. How many brothers and sisters are there?
Ans: 4 boys and 3 girls
22 There is a 5 digit no. 3 pairs of sum is eleven each. Last digit is 3 times the first one.
Third digit is 3 less than
the second. 4th digit is 4 more than the second one. Find the digit.
Ans: 25296
23 There are five thieves, each loot a bakery one after the other such that the first one
takes 1/2 of the total no. of
the breads plus 1/2 of a bread. Similarly 2nd, 3rd, 4th and 5th also did the same. After
the fifth one, no. of
breads remained are 3. Initially how many breads were there?
Ans: 3
24. A software engineer just returned from US, has eaten too much fat & put a lot of
weight. Every Sunday he starts
walking 4 km/hr on level ground, then up at 3 km\hr, then back down hill at km\hr,
then again on level ground at
4km\hr till he reaches his destination. If he returned home at 9 p.m., what distance did
he covered?
Ans: 24 km
25 There are 4 married couples, out of which, 3 poeple in a group is needed. But there
should not be his or her
spouse in the group. How many groups are possible?
Ans: 32
2. In the 4 digits 1,2,3,4, how many 4 digited numbers are possible which are divisible by
4? Repetitions are allowed.
Ans: 64
27. Two men are going along a track of rail in the opposite direction. One goods train
crossed the first person in
20 sec. After 10 min the train crossed the other person who is coming in opposite
direction in 18 sec.
After the train has passed, when the two persons will meet?
Ans: Approx 2 min,
28. At six o'clock the wall clock struck times. Checking with my Watch, I noticed that the
time between the first &
last strokes was 30 Seconds. How long will the clock take to stike 12 at mid night?
Ans: 66 Seconds
29 A person spending out 1/3 for cloths, 1/5 of the remaining for food and 1/4 of the
remaining for travel. He is left
with Rs 100/-. How he had in the beginning?
Ans: Rs 250/-
30. A chain is broken into three pieces of equal lengths, containing 3 links each. It is taken
to a blacksmith to join into
a single continuous one. How many minimum numbers of links are to to be opened to
make it?
Ans: 2
2 Reasoning (30 quest)
Answer Questions to 5 on the basis of the information given below:
Nine individuals - Z, Y, X, W, V, U, T, S and R - are the only candidates, who can serve on
three committees-- A, B and C, and each candidate should serve on exactly one of the
committees.
committee: A should consist of exactly one member more than < name="IL_MARKER"
="">committee B.
It is possible that there are no members of < name="IL_MARKER"
="">committee C.
Among Z, Y and X none can serve on < name="IL_MARKER" ="">committee A.
Among W, V and U none can serve on < name="IL_MARKER"
="">committee G.
Among T, S and R none can serve on < name="IL_MARKER" ="">committee C.
1. In case T and Z are the < name="IL_MARKER" ="">individuals serving on <
name="IL_MARKER" ="">committee B, how many of the nine < name="IL_MARKER"
="">individuals should serve on < name="IL_MARKER" ="">committee C?
1. 3
2. 4
3. 5
4.
5. 7
Ans : B
2. Of the nine < name="IL_MARKER" ="">individuals, the largest number that can serve
together on < name="IL_MARKER" ="">committee C is
1. 9
2. 8
3. 7
4.
5. 5
Ans : D
3. In case R is the only individual serving on < name="IL_MARKER" ="">committee B,
which among the following should serve on < name="IL_MARKER" ="">committee A?
1. W and S
2. V and U
3. V and T
4. U and S
5. T and S
Ans : E
4. In case any of the nine < name="IL_MARKER" ="">individuals serves on <
name="IL_MARKER" ="">committee C, which among the following should be the candidate
to serve on < name="IL_MARKER" ="">committee A?
1. Z
2. Y
3. W
4. T
5. S
Ans : C
5. In case T, S and X are the only < name="IL_MARKER" ="">individuals serving on <
name="IL_MARKER" ="">committee B, the total membership of < name="IL_MARKER"
="">committee C should be:
1. Z and Y
2. Z and W
3. Y and V
4. Y and U
5. X and V
Ans : A
. I am less than feet tall but more than 2 feet tall. My height in inches is a multiple of 7
and is also 2 inches
more than a multiple of . What is my height in inches?
A. 43 B. 5 C. 21 D. 42 E. 5
Ans: B
7. A group of 12 girl scouts had enough food to last for 8 days when they arrived in camp.
However, 4 more scouts
joined them without the amount of food being increased. How long will the food last if
each scout is given the same
daily ration as originally planned?
A. 2 B. 3 C. 4 D. 5 E.
Ans: E
8. A dog takes 3 steps to walk the same distance for which a cat takes 4 steps. Suppose 1
step of the dog covers 1
foot. How many feet would the cat cover in taking 12 steps?
A. 9ft. B. 3ft C. ft D.8ft E. 1ft
Ans: A
9. If a kindergarten teacher places her children 4 on each bench, there will be 3 children
who will not have a place.
However, if 5 children are placed on each bench, there will be 2 empty places. What is
the smallest number of
children the class could have?
A. 33 B. 23 C. 21 D. 24 E. 20
Ans: B
10. The distance around the track is 1000 yards. Each cyclist travels around the track at a
different rate per minute.
For example, Cyclist A travels at 700 yards per minute. At time = 0, Cyclist A is at point
0 (or starting point).
After one minute, they would have traveled 700 yards. After two minutes, they would
have traveled 1400 yards,
but because the track is circular, they would go around again, thus ending them 400
yards after the starting point.
After three minutes, adding 700 again, they would end up 100 yards after the starting
point (because they went
around again). Continue this pattern for all three rates until all three cyclists are at the
same point.
A.10mins B. 11mins C. 21mins D. 9mins E. 20mins
Ans: A
11. If SAVOURY is coded as OVUARSY then how will RADIATE be coded?
(A) AIDARET
(B) IDARA TE
(C) ARIADTE
(D) IDAATRE
(E) None of these
Ans : (D)
12. If MAPLE is coded as VOKZN then how will CAMEL be coded?
(A) OVNZF
(B) OUNZX
(C) OVNZX
(D) XZNVO
(E) None of these
Ans : (C)
13. If CRY is coded as MRYC then how will GET be coded?
(A) MTEG
(B) MGET
(C) MEGT
(D) METG
(E) None of these
Ans : (D)
14. If BURNER is coded as CASOIS then how will ALIMENT be coded?
(A) BKJLFMU
(B) EKOLIMS
(C) EMONIOU
(D) BRJSFTU
(E) EROSITU
Ans : (C)
15. If Sand is coded as Brick, Brick as House, House as Temple, Temple as Palace then
where do you worship?
(A) Palace
(B) Temple
(C) Brick
(D) House
(E) None of these
Ans : (A)
1. How many such letter-pairs are there in the word SERVANT having the same no. of
letters left between them in
the word as they have in the series?
(A) 2
(B) 3
(C) 4
(D) 5
(E) None of these
Ans : (A)
17. How many such letter-pairs are there in the word MONKEY having same no. of letters
left between them as they
have in the series?
(A) 3
(B) 4
(C) 1
(D) 5
(E) None of these
Ans : (C)
18. How many such letter-pairs are there in the word SMUGGLER having same no. of letters
left between them as
they have in the series?
(A) 2
(B) 3
(C) 4
(D) 1
(E) None of these
Ans : (A)
19. How many such letter-pairs are there in the word BONAFIDE having same no. of letters
left between them as
they have in the series?
(A) 2
(B) 3
(C) 4
(D) 1
(E) None of these
Ans : (E)
20. How many such letter-pairs are there in the word FRONTIER having same no. of letters
left between them as they have in the series?
(A) 2
(B) 4
(C) 1
(D) 3
(E) None of these
Ans : (A)
Directions 2-25 : In each question below are given two statements followed by
two conclusions numbered I and II You have to take the given two statements to
be true even if they seem to be at variance from commonly known facts Read the
conclusions and then decide which of the given conclusions logically follows from
the two given statements, disregarding commonly known facts
ive answer (A) if only conclusion I follows; (B) if only conclusion II follows; (C) if
either I or II follows (D) if neither I or II follows and (E) if both I and II
21.Statements : Some soldiers are famous
Some soldiers are intelligent
Conclusions : I. Some soldiers are either famous or intelligent
II. Some soldiers are neither famous nor intelligent
Ans: D
22.Statements : All boys are honest
Sachin is honest
Conclusions : I. Sachin is a boy
II. All honest persons are boys
Ans: D
23.Statements : Some nurses are nuns
Madhu is a nun
Conclusions : I. Some nuns are nurses
II. Some nurses are not nuns
Ans: D
24.Statements : All windows are doors
No door is wall
Conclusions : I. No window is wall
II. No wall is door
Ans: A
25.Statements : All poles are guns
Some boats are not poles
Conclusions : I. All guns are boats
II. Some boats are not guns
Ans: D
2. How many such pairs of letters are there in the word GUARDIAN each of which has as
many letters between
them in the word as in the English alphabet ?

(A) None
(B) One
(C) Two
(D) Three
(E) More than three
Ans: D

27. In a certain code language `pik da pa means `where are you; `da na ja means `you
may come and `na ka sa means `he may go, which of the following means `come in that
code language ?

(A) da
(B) ja
(C) na
(D) Cannot be determined
(E) None of these
Ans: B
28. What should come next in the following number series ? 9 8 9 8 7 9 8 7 9 8 7 5 9
8 7 5 4 9 8 7 5
(A) 3
(B) 4
(C) 2
(D) 1
(E) None of these
Ans: B
29. Meeta correctly remembers that her fathers birthday is after 8th July but before 12th
July. Her brother correctly
remembers that their fathers birthday is after 10th July but before 15th July. On which
day of July was definitely
their fathers birthday ?
(A) 10th
(B) 11th
(C) 10th or 11th
(D) Cannot be determined
(E) None of these
Ans: B
30. Four of the following five are alike in a certain way and so form a group. Which is the
one that does not belong to that group ?
(A) B D F
(B) V X Z
(C) F I K
(D) M O Q
(E) L N P
Ans: C
3 English (30 quest)
Directions for Questions to 5: The passage given below is followed by a set of
five questions Choose the most appropriate answer to each question
A remarkable aspect of art of the present century is the range of concepts and ideologies
which it embodies. It is almost tempting to see a pattern emerging within the art field - or
alternatively imposed upon it a posteriori - similar to that which exists under the umbrella
of science where the general term covers a whole range of separate, though
interconnecting, activities. Any parallelism is however - in this instance at least -
misleading. A scientific discipline develops systematically once its bare tenets have been
established, named and categorized as conventions. Many of the concepts of modern art, by
contrast, have resulted from the almost accidental meetings of groups of talented
individuals at certain times and certain places. The ideas generated by these chance
meetings had two - fold consequences. Firstly, a corpus of work would be produced which,
in great part, remains as a concrete record of the events. Secondly, the ideas would
themselves be disseminated through many different channels of communication - seeds
that often bore fruit in contexts far removed from their generation. Not all movements were
exclusively concerned with innovation. Surrealism, for instance, claimed to embody a kind
of insight which can be present in the art of any period. This claim has been generally
accepted so that a sixteenth century painting by Spranger or a mysterious photograph by
Atget can legitimately be discussed in surrealist terms. Briefly, then, the concepts of
modern art are of many different (often fundamentally different) kinds and resulted from
the exposures of painters, sculptors and thinkers to the more complex phenomena of the
twentieth century, including our ever increasing knowledge of the thought and products of
earlier centuries. Different groups of artists would collaborate in trying to make sense of
rapidly changing world of visual and spiritual experience. We should hardly be surprised if
no one group succeeded completely, but achievements, through relative, have been
considerable. Landmarks have been established - concrete statements of position which
give a pattern to a situation which could easily have degenerated into total chaos. Beyond
this, new language tools have been created for those who follow - semantic systems which
can provide a springboard for further explorations.

The codifying of art is often criticized. Certainly one can understand that artists are wary of
being pigeon- holed since they are apt to think of themselves as individuals - sometimes
with good reason. The notion of self-expression, however, no longer carries quite the weight
it once did; objectivity has its defenders. There is good reason to accept the ideas codified
by artists and critics, over the past sixty years or so, as having attained the status of
independent existence - an independence which is not without its own value. This time
factor is important here. As an art movement slips into temporal perspective, it ceases to be
a living organism - becoming, rather, a fossil. This is not to say it becomes useless or
uninteresting. Just as a scientist can reconstruct the life of a prehistoric environment from
the messages codified into the structure of a fossil, so can an artist decipher whole webs of
intellectual and creative possibility from the recorded structure of a `dead art movement.
The artist can match the creative patterns crystallized into this structure against the
potentials and possibilities of his own time. AS T.S Eliot observed, no one starts anything
from scratch; however consciously you may try to live in the present, you are still involved
with a nexus of behaviour patterns bequeathed from the past. The original and creative
person is not someone who ignores these patterns, but someone who is able to translate
and develop them so that they confirm more exactly to his - and our - present needs.
1. Many of the concepts of modern art have been the product of
(1) ideas generated from planned deliberations between artists, painters and thinkers.
(2) the dissemination of ideas through the state and its organizations.
(3) accidental interactions among people blessed with creative muse.
(4) patronage by the rich and powerful that supported art
(5) systematic investigation, codification and conventions.
Ans 3
2. In the passage, the word `fossil can be interpreted as
(1) an art movement that has ceased to remain interesting or useful.
(2) an analogy from the physical world to indicate a historic art movement.
(3) an analogy from the physical world to indicate the barrenness of artistic creations in the
past.
(4) an embedded codification of pre-historic life.
(5) an analogy from the physical world to indicate the passing of an era associated with an
art movement.
Ans 5
3. In the passage, which of the following similarities between science and art may lead to
erroneous conclusions?
(1) Both, in general, include a gamut of distinct but interconnecting activities.
(2) Both have movements not necessarily concerned with innovation.
(3) Both depend on collaborations between talented individuals.
(4) Both involve abstract thought and dissemination of ideas.
(5) Both reflect complex priorities of the modern world.
Ans

4. The range of concepts and ideologies embodied in the art of the twentieth century is
explained by
(1) the existence of movements such as surrealism.
(2) landmarks which give a pattern to the art history of the twentieth century.
(3) new language tools which can be used for further explorations into new areas.
(4) the fast changing world of perceptual and transcendental understandings.
(5) the quick exchange of ideas and concepts enabled by efficient technology.
Ans 4
5. The passage uses an observation by T.S. Eliot to imply that
(1) creative processes are not `original because they always borrow from the past.
(2) we always carry forward the legacy of the past.
(3) past behaviours and thought processes recreate themselves in the present and get
labeled as `originalorcreative.
(4) `originality can only thrive in a `green house insulated from the past biases.
(5) `innovations and `original thinking interpret and develop on past thoughts to suit
contemporary needs.
Ans 5
Directions 6:0 Which of the phrases (A), (B), (C) and (D) given below each sentence
should replace the phrases printed in bold in the following sentences to make the sentence
grammatically correct. If the sentence is correct as it is and no correction is required, mark
(E) as the answer.

. Her face buries itself on his rough shirt and he could feel the fragrance of her hair and
the warmth of her as she sobbed against his breast-
(A) sinking itself in his rough shirt
(B) buried itself in his rough shirt
(C) dipped itself in his rough shirt
(D) sank itself in his rough shirt
(E) No correction required
Ans: B

7. `Allah, he said, rising his face towards the star-spangled black sky, "punish me as
much as you like-Mahesh died with thirst on his lips.
(A) raised his face towards the star-spangled black sky
(B) had raised his face towards the star-spangled black sky
(C) rose his face towards the star spangled black sky
(D) raising his face towards the star-spangled black sky
(E) No correction required
Ans: D

8. All of us carried plenty of food and sweets with us and we served the little fortunate
human beings with our own hands-
(A) The lesser fortunate human beings
(B) The less fortunate human beings
(C) The least fortunate human beings
(D) The unfortunate human beings
(E) No correction required
Ans: B

9. Once again, the Indian tradition of `ahimsa comes out as infinitely most relevant, than
much of what we learn in modern education.
(A) The most relevant, than much of what we learn
(B) More relevance what we learn
(C) More relevant than much of what we learn
(D) No relevance in what we learn
(E) No correction required
Ans: C

10. When it was decided to send the rescue team in the colliery, the experts showed their
reluctance.
(A) into the colliery
(B) inside the colliery
(C) under the colliery
(D) underneath the colliery
(E) No correction required
Ans: A
Directions(-5): In each of the following questions, there is a certain
relationshipbetween two
given words on one side of : : and one word is given on another side of : :while
another word is to be found from the given alternatives, having the same relation
with this word as the words of the given pair bear Choose the correct
alternative
11. Appraiser : Building : : Critic : ?
(A) Book (B) Masterpiece (C) Judge (D) Gold
Ans: (A)
12. Tile : Mosaic : : Knot : ?
(A) Embroidery (B) Abacus (C) Macrame (D) Easle
Ans: (C)
13. Bread : Yeast : : Curd : ?
(A) Fungi (B) Bacteria (C) Germs (D) Virus
Ans: (B)
14. Command : Order : : Confusion : ?
(A) Discipline (B) Clarity (C) Choas (D) Problem
Ans: (C)
15. Ruby : Red : : Sapphire : ?
(A) Blue (B) White (C) Green (D) Silver
Ans: (A)
Directions 6 to 20: Pick out from the words, given below each sentence, the
word which would complete the sentence correctly and meaningfully
1.Usually the ascent of a mountain face is much easier than the...................
A) fall B) decent C) descent D) descend
Ans: C
17. More than twenty years have now passed....................I had my first flight.
A) when B) since C) while D) as
Ans: B
18. The river overflowed its................... and flooded the area
A) Edges B) Fronts C) Limits D) Banks
Ans: D
19. Once you suspect a person of double dealing, you ought to keep him at arm's.
A) distance B) length C) aim D) width
Ans: B
20. While strolling on Janpath, I chanced to meet..............European.
A) one B) the C) an D) a
Ans: D
Directions for Questions 2 to25 :Each of the following sentences has been divided
into four parts There is an error in one of the parts Point out the part which has
an error

21. A) he has broken his pen, /
B) and his paper is not yet complete, /
C) but you can help him /
D) if you give him your for an hour.
Ans: D
22. A) This book is quite different /
B) than the one I gave you, /
C) because it is an old edition /
D) and does not cover the whole course.
Ans: B
23. A) He could come /
B) if you invite him /
C) well in time /
D) in writing.
Ans: B
24. A) Had, the doctor reached an hour earlier /
B) and gave him proper medicine, /
C) his life would have been saved /
D) without any doubt.
Ans: B
25. A) He ought have tried his best /
B) from the very beginning /
C) because the goal he set before him /
D) was not an ordinary one.
Ans: D
Directions 26-30 : In each of the following questions, out of the given alternatives,
choose the one which is nearest in meaning to the given word
2. ERROR
A) Blunder B) Nisadventure C) Ambiguity D) Misgiving
Ans: A
27. ACRID
A) Dirty B) Pungent C) Unripe D) Bitter
Ans: D
28. REQUITE
A) Servile B) Trashy C) Ferocious D) Juicy
Ans: B
29. BADER
A) Attempt B) Convince C) Pester D) Persuade
Ans: C
30. CAUTIOUSLY
A)Genuinely B) Carefully C) Secretly D) Somewhat.
Ans: B
L&T Q & A-II

WRITTEN TEST
Total 90 questions , 90 minutes Offline (paper & pen) test---SECTIONAL CUTOFF IS
THERE
1. Aptitude (30 quest)
2. Reasoning (30 quest)
3. English (30 quest)
Aptitude (30 quest)
1. If he sells 40 mangoes, he will get the selling price of 4 mangoes extra, What is his %
increase in profit ?
Ans: 25%
2. Two pipes can fill a tank in 10 and 12 hours respectively while third pipe will make the
tank empty in 20
hours. If all three pipes operate simultaneously, in how many hours the tank will be
filled ?

Anshours 30 minutes
3. Cost of an item is x. It's value increases by p% and decreases by p% Now the new value
is 1rupee,
what is the actual value ?

Ans(000)/(000-p*p)
4. 1215 is the volume of some material. How many cubes of edge 3 can be inserted into
it ?

Ans40

5. 100 glasses are there. A servant has to supply glasses to a person If he supplies the
glasses without
any damage he will get 3 paise otherwise he will loose 3 paise. At the end of supplying
100 glasses if
he gets 270 paise, how many glasses were supplied safely.

Ans 95
. In a class composed of x girls and y boys what part of the class is composed of girls ?

A. y/(x + y)
B. x/xy
C. x/(x + y)
D. y/xy

Ans C

7. What is the maximum number of half-pint bottles of cream that can be filled with a 4-
gallon can
of cream(2 pt.=1 qt. and 4 qt.=1 gal)

A.1
B.24
C.30
D.4

AnsD

8. If the operation,^ is defined by the equation x ^ y = 2x + y, what is the value of a in 2 ^
a = a ^ 3

A. 0
B. 1
C. -1
D. 4

AnsB
9. A coffee shop blends 2 kinds of coffee, putting in 2 parts of a 33p. a gm. grade to 1 part
of a 24p. a gm.
If the mixture is changed to 1 part of the 33p. a gm. to 2 parts of the less expensive
grade, how much
will the shop save in blending 100 gms.

A.Rs.90
B.Rs.1.00
C.Rs.3.00
D.Rs.8.00

AnsC

10. There are 200 questions on a 3 hr examination. Among these questions are 50
mathematics problems.
It is suggested that twice as much time be spent on each maths problem as for each
other question.
How many minutes should be spent on mathematics problems

A. 3
B. 72
C. 0
D. 100

AnsB

11. In a group of 15,7 have studied Latin, 8 have studied Greek, and 3 have not studied
either. How many of
these studied both Latin and Greek

A.0
B.3
C.4
D.5

AnsB

12. In June a baseball team that played 0 games had won 30% of its game played. After a
phenomenal
winning streak this team raised its average to 50% .How many games must the team
have won in a
row to attain this average?
A. 12
B. 20
C. 24
D. 30
Ans C
13. M men agree to purchase a gift for Rs. D. If three men drop out how much more will
each have to
contribute towards the purchase of the gift ?
A. D/(M-3)
B. MD/3
C. M/(D-3)
D. 3D/(M
2
-3M)
Ans D
14. A company contracts to paint 3 houses. Mr. Brown can paint a house in days while Mr.
Black would
take 8 days and Mr. Blue 12 days. After 8 days Mr. Brown goes on vacation and Mr.
Black begins to
work for a period of days. How many days will it take Mr. Blue to complete the
contract?
A. 7
B. 8
C. 11
D. 12

AnsC
15. 2 hours after a freight train leaves Delhi a passenger train leaves the same station
traveling in the
same direction at an average speed of 1 km/hr. After traveling 4 hrs the passenger
train overtakes
the freight train. The average speed of the freight train was?
A. 30
B. 40
C.58
D. 0
Ans B

1. A fold density is 19 times greater than the water and for copper it is 9 times. At what
ratio you can mix gold
and copper to get 15 times denser than water. ?
Ans: 3 : 2
17. There is a room with ' x 8'. A 1' tile is fixed along the 4 walls in one row. How many 1"
tiles require to
finish the work. ?
Ans: 24
18. 2 persons can finish a job in 8 days. First person alone can finish the work in 24 days.
How many days
does the second person take to finish the job?
Ans: 2 days

19. In a class total 34 students, 1 are have a brother, 15 are have sisters, 9 students don't
have either brothers
or sisters. Find the number of students having both brother and sisters.
Ans: 6
21. In a car wheel, two spokes cover 15 degree. Then for the entire car, how many spokes
are there?
Ans: 24
20. What is the angle of degree suspended when two hands of clock showing the time 2:30?
Ans: 05 degrees
21. The age difference between two brothers is 3 years. After years the ratio between the
age is 9:8. What are their ages?
Ans: 2 and 8
22. A person's salary is getting reduced by 20%. What percentage should be added to get
back his original salary?
Ans: 25%
23. Two persons start at the same point, walk in opposite directions with 5 km/hr and 5.5
km/hr respectively.
What is the distance separated after 2 and half hrs?
Ans: 2625 (approx)
24. A person starts walking at a speed of 5 km/hr through half the distance, rest of the
distance he covers with
a speed 4km/hr. Total time of travel is 9 hours. What is the maximum distance he can
cover?
Ans: 40km
25. Initially two cups of same volume are present with milk filled upto 3/5th and 4/5th of
their volumes.
Water is then filled. Then two mixtures are mixed. Find the ratio of water to milk in the
mixture.
Ans: 3 :
2. 1 grams of radioactive material decays into 8 grams in 10 years. How long will it take
to decay to 1 gram?
Ans: 0 yrs
27. In a rectangle the length is increased by of the original length. By what proportion
should the width be reduced
so that the area will be the same?
Ans: 33
28. If a square is formed by the diagonal of the square as an edge, what is the ratio
between the area?
Ans: 2
29. The perimeter of a rhombus is 52 units. One of its diagonal is 24 units. What is its
second diagonals length?
Ans: 0
30. A batsman scored 18 runs in his 18th innings and that makes his average 18. Find his
average up to the 17th
innings?
Ans: 9
2 Reasoning (30 quest)
1. One of the following is my secret word: AIM DUE MOD OAT TIE. With the list in front of
you, if I were
to tell you any one of my secret word, then you would be able to tell me the number of
vowels in my secret word.
Which is my secret word?
Ans TIE
2. One of Mr. Horton, his wife, their son, and Mr. Horton's mother is a doctor and another is
a lawyer.
a) If the doctor is younger than the lawyer, then the doctor and the lawyer are not blood
relatives.
b) If the doctor is a woman, then the doctor and the lawyer are blood relatives.
c) If the lawyer is a man, then the doctor is a man. Whose occupation you know?

Ans Mr Horton: he is the doctor
3. Three piles of chips--pile I consists one chip, pile II consists of chips, and pile III consists
of three
chips are to be used in game played by Anita and Brinda. The game requires:
a) That each player in turn take only one chip or all chips from just one pile.
b) That the player who has to take the last chip loses.
c) That Anita now have her turn. From which pile should Anita draw in order to win?

Ans Pile II
4. Mr. and Mrs. Aye and Mr. and Mrs. Bee competed in a chess tournament. Of the three
games played:
a) In only the first game were the two players married to each other.
b) The men won two games and the women won one game.
c) The Ayes won more games than the Bees.
d) Anyone who lost game did not play the subsequent game. Who did not lose a game?

Ans MrsBee
5. II A to Z is written in the reverse order which is the 5th letter to the right oI the 11th letter
Irom the leIt ?
ns . K

Directions 6-0 The office staff of XYZ corporation presently consists of three
bookkeepers A, B, C and 5 secretaries D, E, F, G, H. The management is planning to open a
new office in another city using 2 bookkeepers and 3 secretaries of the present staff . To do
so they plan to separate certain individuals who don't function well together. The following
guidelines were established to set up the new office
I. Bookkeepers A and C are constantly finding fault with one another and should not be sent
together to the new office as a team
II. C and E function well alone but not as a team , they should be separated
III. D and G have not been on speaking terms and shouldn't go together
IV Since D and F have been competing for promotion they shouldn't be a team
. If A is to be moved as one of the bookkeepers, which of the following cannot be a
possible working unit.

A. ABDEH
B. ABDGH
C. ABEFH
D. ABEGH

AnsB


7. If C and F are moved to the new office, how many combinations are possible

A. 1
B. 2
C. 3
D. 4

AnsA

8. If C is sent to the new office, which member of the staff cannot go with C

A. B
B. D
C. F
D. G

AnsB

9. Under the guidelines developed, which of the following must go to the new office

A. B
B. D
C. E
D. G

AnsA

10. If D goes to the new office, which of the following is/are true

I. C cannot go
II. A cannot go
III. H must also go

A. I only
B. II only
C. I and II only
D. I and III only

AnsD
11.If Leader is coded as Actor, Actor as an Administrator, Administrator as a Traitor, Traitor
as a Social-reformer,
then who takes part in the Parliamentary elections?
(A) Administrator
(B) Leader
(C) Traitor
(D) Social-Reformist
(E) None of these
Ans : (E)
12. If '245' means-'Art and Talent' in a certain code language, '31' means- 'Callous to
Generous', '147' means-
'Callous and Polite' then what is the code used for 'to' ?
(A) Only 3
(B) Only 1
(C) 3 or
(D) Only
(E) None of these
Ans : (C)
13. In a certain code language '579' means-'Kanchan is soft-spoken', '94' means-`Soft-
spoken beautiful pure',
'473' means-`Ganga is pure', then what is the code used for 'Kanchan' ?
(A) 7
(B) 5
(C) 9
(D) Can not be determined
(E) None of these
Ans : (B)
14. Which of the following relates to FLOWER in the same way as RTERBN relates to
SECTOR?
(A) RWLGPF (B) EOFKUQ (C) EOFMXS
(D) RWLEPD (E) RWLEND
Ans (E)
15. ,ir is cloud, cloud is r,in, r,in is w,ter, w,ter is s,nd so wh,t is cloud?
ns :sand

Directions ( questions 6 to 20 ): Read the following information carefully and
answer the questions given below :
()A, B, C, D and E are five friends
(2)B is elder to E, but not as tall as C
(3)C is younger to A, and is taller to D and E
(4)A is taller to D, But younger to E
(5)D is elder to A but is shorter in the group
1. Who among the following is the eldest?
A). A B). B C). C D). D E). None of these
Ans: E
17. Which of the following pairs of students is elder to D?
A) BA B). BC C). BE D) EA E) None of these
Ans: E
18. Which of the following statements is correct about B?
1) B is not the tallest 2) B is shorter to E
(3)When they are asked to stand in ascending order with respect to their heights, B is in
the middle
A) Only (1) is correct B) Only (1) and (3) are correct C) All are correct
D) All are incorrect E) None of these
Ans: A
19. If F, another friend is taller than C, how many of them will be between F and E
according to their height?
A) None B) One C) Two D) Three E) None of these
Ans: E
20. If a selection is to be made among them who would be relatively older and also taller,
who among them
should be chosen?
A) A B) B C) C D) D E) E
Ans: B
Directions (questions 2 to 25): Each of the questions below consists of a question
and two statements marked I and II given below it You have to decide whether
the data provided in the statements are sufficient to answer the question Read
both the statements and give answer
(A) If the data in statement I alone are sufficient to answer the question, while
the data in statement II alone are not sufficient to answer the question (B) If the
data in statements I alone are not sufficient to answer the question, while the
data in statement II alone are sufficient to answer the question (C) If the data
either in statement I or in statement II alone are sufficient to answer the
question (D) If the data even in both the statements I and II together are
not sufficient to answer the question (E) If the data in both statements I and II
together are needed to answer the question
21.. What does 'Ne' stands for in the code language?
I. 'Na Ni Nok Ne' means 'I will tell you' and 'Ni Nok Ne Nam' means 'he will tell you' in
that code language.
II. 'Ni Ne Mo Nam' means 'will he call you' and 'Ne Mok Sac Ni' means 'how will you go' in
that code language.
Ans (D)
22. Who amongst P, Q, R, S, T and U is the tallest?
I. P is taller that R and T but not as tall as U, who is taller than Q and S.
II. R is the third in height in the ascending order and not as tall as U, P and Q, Q being
taller than P but not the
tallest.
Ans (C)
23. Who among A, B, C, D, E & F read the book last?
I. F, who gave the book to B after reading, was third to read the same.
II. C, who read the book after A, was the third person to read the book before it reached
E.
Ans (D)
24. Who is paternal uncle of Pavan?
I. Pavan is brother of Poornima, who is daughter of Meena, who is sister of Kumar, who
is brother Smrithi.
II. Prithvi is brother of Indrajith, who is husband of Poornima, who is mother of Ganga,
who is sister of Pavan.
Ans (B)
25. What is Milan's rank in the class of 44 students?
I. Ramesh, whose rank is 17th in the class, is ahead of Shyam by ranks, Shyam being
7 ranks ahead of Milan.
II. Suketu is 2 ranks ahead of Milan and Shyamala is ranks behind Milan while Savita
stands exactly in the
middle of Shyamala and Suketu in ranks, her rankbeing 17.
Ans (c)
Directions 26-2: Each question consists of five statements followed by options
consisting of three
statements put together in a specific order Choose the option
which indicates a
valid argument, ie where the third statement is a conclusion
drawn from the
preceding two statements
2. A. All software companies employ knowledge workers.
B. Tara Tech employs knowledge workers.
C. Tara Tech is a Software company.
D. Some software companies employ knowledge workers.
E. Tara Tech employs only knowledge workers.

A) ABC B) ACB C) CDB D) ACE
Ans: B
27. A. Traffic congestion increases carbon monoxide in the environment.
B. Increase in carbon monoxide is hazardous to health.
C. Traffic congestion is hazardous to health.
D. Some traffic congestion does not cause increased carbon monoxide.
E. Some traffic congestion is not hazardous to health.

A. CBA B. BDE C. CDE D. BAC
Ans: D
28. All scientists are fools. All fools are literates.

A. All literates are scientists
B. All scientists are literates
C. No scientists are literates
D. Both (a) and (b) are correct

Ans B

29. All shares are debentures. No debentures are deposits.



A. All shares are deposits
B. Some shares are deposits
C. No shares are deposits
D. None of the above

Ans C

30. Some green are blue. No blue are white.



A. No green are white
B. Some green are white
C. No green are white
D. None of the above

Ans B

3 English (30 quest)

Directions -5
To those who do listen, the desert speaks of things with an emphasis quite different from
that of the shore, the mountain, the valley or the plains, whereas these invite action and
suggest limitless opportunity and exhaust less resources, the implications and the mood of
the desert are something different. For one thing , the desert is conservative not radical. It
is more likely to provide awe than to invite conquest. The heroism which it encourages is
the heroism of endurance, not that of conquest. It brings man up against this limitation,
turns him upon himself and suggests values which more indulgent regions suppress.
Sometimes it includes contemplation in men who have never contemplated before : And of
all the answers to the question- what is a desert good for - 'contemplation' is perhaps the
best.
1. In order to receive the desert's message, the beholder needs to be
a) courageous in his reaction.
b) conservative in his responses.
c) A good listener.
d) Sensitive to nature.
Ans :D
2. The desert is unique among landscapes in that it encourages only
a) contemplation
b) Indolence
c) Heroic Endeavor
d) Adventurous Spirit
Ans :A
3. If one responds with insight to the mood of the desert, it evokes
a) An inclination for deep thought
b) The possibility of unending resources
c) The desire for Heroic conquest
d) A sense of intense revulsion
Ans :A
4. The writer calls the desert "conservative rather than radical because it provides an
environment that
a) Inspires man to explore it
b) Offers unlimited opportunity to conquer
c) Tests ones Endurance
d) makes one gloomy
Ans :C
5. What does the phrase it brings man up against his limitations, mean?
a) It makes man feel hopeless about his limitations
b) It makes man aware of his limitations
c) It compels man to fight against his limitations
d) It persuades man to overcome his limitations
Ans :D
Directions 6-0: In each of the following questions, there is a certain
relationshipbetween two given words on one side of : : and one word is given on
another side of : :while another word is to be found from the given alternatives,
having the same relation with this word as the words of the given pair bear
Choose the correct alternative

. Errata : Books : : flaws:?
(A) Manuscripts (B) Metals (C) Speech (D) Charter
Ans: (B)
7. Planet : Orbit : : Projectile : ?
(A) Trajectory (B) Track (C) Milky way (D) Path
Ans: (A)
8. Appraiser : Building : : Critic : ?
(A) Book (B) Masterpiece (C) Judge (D) Gold
Ans: (A)
9. King : Throne : : Rider : ?
(A) Seat (B) Horse (C) Saddle (D) Chair
Ans: (C)
10. Reluctant : Keen : : Remarkable : ?
(A) Usual (B) Restrained (C) Striking (D) Evolution
Ans: (A)
Directions -5: In each of the following questions, out of the given alternatives,
choose the one which best expresses the meaning of the given word
11 BEMOAN
A. Lament B. Soothe C. Denounce D. Loathe
Ans: A
12 ABRADE
A. Scold B. attach C. Wear off D. Twist
Ans: C
13. DISTINUISH
A. Darken B. Abolish C. Differentiate D. Confuse
Ans: A
14 PORTRAY
A. Communicate B. Paint C. Express D. Draw
Ans: D
15 INFINITE
A. Strange B. Endless C. Indefinite D. Vague
Ans: B

Directions 6-: Read each sentence to find if there is any grammatical error in
it If there is any error, it will be only one part of the sentence The number or
alphabet of that part is your answer
1. I shall / ring him / tomorrow / in the afternoon.
A B C D

Ans: B
17. I enjoyed / during my / stay in / England.
A B C D

Ans: A
18. I believe / that respect / is more preferable than / money.
A B C D

Ans: C
19. I never have / visited / or intend to visit / foreign countries /
A B C D

Ans: C
20. In this way nuclear fission / or the splitting / of the atom / have been achieved /
A B
L&T Q & A-III

WRITTEN TEST
Total 90 questions , 90 minutes ---SECTIONAL CUTOFF IS THERE
1. Aptitude (30 quest)
2. Reasoning (30 quest)
3. English (30 quest)
Aptitude (30 quest)
1. Three friends divided some bullets equally. After all of them shot 4 bullets the total
number of bullets remaining is equal to the bullets each had after division. Find the original
number divided.
Ans: 8
2. In , cl,ss composed oI x girls ,nd y boys wh,t p,rt oI the cl,ss is composed oI girls

A.y/(x y)
B.x/xy
C.x/(x y)
D.y/xy

ns.C

3 What is the maximum number of half-pint bottles of cream that can be filled with a 4-
gallon can of cream(2 pt.=1 qt. and 4 qt.=1 gal)

A.1
B.24
C.30
D.4

AnsD

4. If the operation,^ is defined by the equation x ^ y = 2x + y, what is the value of a in 2 ^
a = a ^ 3

A.0
B.1
C.-1
D.4

AnsB

5. A coffee shop blends 2 kinds of coffee,putting in 2 parts of a 33p. a gm. grade to 1 part
of a 24p. a gm.If the mixture is changed to 1 part of the 33p. a gm. to 2 parts of the less
expensive grade,how much will the shop save in blending 100 gms.

A.Rs.90
B.Rs.1.00
C.Rs.3.00
D.Rs.8.00
AnsC

. There are 200 questions on a 3 hr examination.Among these questions are 50
mathematics problems.It is suggested that twice as much time be spent on each maths
problem as for each other question.How many minutes should be spent on mathematics
problems

A.3
B.72
C.0
D.100

AnsB

7. In a group of 15,7 have studied Latin, 8 have studied Greek, and 3 have not studied
either.How many of these studied both Latin and Greek

A.0
B.3
C.4
D.5

AnsB
8. .If 13 = 13w/(1-w) ,then (2w)2 =

A.1/4
B.1/2
C.1
D.2

AnsC
9. In june a baseball team that played 0 games had won 30% of its game played. After a
phenomenal winning streak this team raised its average to 50% .How many games must
the team have won in a row to attain this average?A. 12
B. 20
C. 24
D. 30
Ans C
10. A company contracts to paint 3 houses. Mr.Brown can paint a house in days while
Mr.Black would take 8 days and Mr.Blue 12 days. After 8 days Mr.Brown goes on vacation
and Mr. Black begins to work for a period of days. How many days will it take Mr.Blue to
complete the contract?
A. 7
B. 8
C. 11
D. 12

AnsC
11. 2 hours after a freight train leaves Delhi a passenger train leaves the same station
travelling in the same direction at an average speed of 1 km/hr. After travelling 4 hrs the
passenger train overtakes the freight train. The average speed of the freight train was?A. 30
B. 40
C.58
D. 0
Ans B
12 x,y ,nd z ,re odd no. not necess,rly in sequence.then y?
(1)me,n oI x ,nd z is 114.
(2)me,n oI y ,nd z is 113.

ns: 115


13. r,m is 27 ye,r old then moh,n. ,Iter 7 ye,r r,m ,ge is thrice ,s moh,n ,ge .Iind r,m ,ge?
ns : 67/2

14 if three tapes are filling a tank of capacity of 500lit with speed of 30lit/sec,48lit/sec and
3lit/sec.
Find after how long time tank will fill?
Ans : 500/4
15. suppose in a pot 20 cards are there label 1-20 on that. find the probability of getting
two prime.

Ans : 8/20 = 2 / 5

1. If a person sells a product for rs141/- he suffers a loss of %.if he has to have a profit
of 10%,
at what price should he sell it?
(a) rs250
(b) rs.175
(c) rs 195.
(d) rs.15
Ansrs65
17. A ball falls from a height of 8ft ,bounces back to half the distance & continues till it
comes to rest.
what is the total distance traveled by the ball?
(a) 24ft
(b)30 ft
(c) infinite
(d) cannot be determined
Ans:(a)

18. If 3 houses are to be painted,mr A can paint a house in days(nos are not same)...mr
B can do the same in 8
days...& mr.C in 12 days.if mr A does the work for 8 days & leaves for vacation, & mr B
continues the work for
the next days, for how many days should mr.C work?
Ans: days(check)
19. one student takes 20 mins to reach school if he goes at a speed of 15kmph .At
what speed should he go to
reach in 15 mins ( nos are not same)
Ans: 6kmph
20. How many rational numbers are there between 0 & 5 ?
Ans:infinite
21. A cube of 3 unit has surface is painted. If u cut the cube of 1 unit each how many
cubes remain unpainted
on all sides ?
Ans

22. one questions in profit and loss a person sold an article at Rs.141 at % loss, to gain a
profit of 10% at what price
the article should be sold.
Ans: 65 Rs
23. 1st day of the year 1999 is sunday and what will be last day of the same year
Ans: sunday
24. squares of equal size are placed side by side to form a rectangle whose perimeter is
182. find the perimeter of
the single square.
Ans: 52

25 .The average age of students in a class is 1.when a teacher of age 40 is added , the
average becoems 17. what is
the total number of students
Ans23
2.One persont travlled 4 laps with the speed of 10,20,40 ( or 30) and 0 kmph and what is
the average speed.
Ans: 20
27 If Rs20/- is available to pay for typing a research report & typist A produces 42 pages
and typist B produces
28 pages. How much should typist A receive?
Ans: :Rs2/-
28. 4 thieves rob a bakery of the bread one after the other. each thief takes half of what is
present ,& half
a bread...if at the end 3 bread remains, what is the no of bread that was present
initially?
Ans:63
29. Six squares of same dimension are kept side by side to make a rectangle with a
perimeter of 182cm.
what is the perimeter of each square ?
Ans: 52
29. one student takes 20 mins to reach school if he goes at a speed of 15kmph .At
what speed should he go to
reach in 15 mins ( nos are not same)
Ans: 1kmph( check)

30. A clerk multiplied a number by ten when it should have been divided by ten. The ans he
got was 100.what should
the answer have been?
Ans::

2 Reasoning (30 quest)
Answer Questions to 5 on the basis of the information given below:
There are three on-off switches on a control panel A, B, and C. They have to be changed
from an initial setting to a second setting according to the following conditions : In case only
switch A is the switch on in the < name="IL_MARKER" ="">initial setting , then turn on
switch B.
In case switches A and B are the only switches on in the < name="IL_MARKER"
="">initialsetting, then turn on switch C. In case all the three switches are on initially
setting, then turn off < name="IL_MARKER" ="">the switch C. For any other <
name="IL_MARKER" ="">initial setting, turn on all switches that are off and turn off all
switches, if any, that are on.
1. In case in the < name="IL_MARKER" ="">initial setting is the switches <
name="IL_MARKER" ="">A and B are on and < name="IL_MARKER" ="">the switch C is
off, then what could be the second setting?
1. A on, B on, C on.
2. A on, B off, C on.
3. A on, B off, C off.
4. A off, B on, C off.
5. A off, B off, C on.
Ans : A
2. In case switch B is the only switch on in the < name="IL_MARKER" ="">initial setting,
what must be the second setting?
1. A on, B on, C on.
2. A on, B on, C off.
3. A on, B off, C on.
4. A off, B off, C on.
5. A off, B off, C off.
Ans : C
3. In case all the three switches are on in the second setting, which among the following
could have been the < name="IL_MARKER" ="">initial setting ?
1. A on, B on, C on.
2. A on, B on, C off.
3. A on, B off, C on.
4. A on, B off, C off.
5. A off, B on, C off
Ans : B
4. In case switch A is off in the second setting, which among the following could have been
the < name="IL_MARKER" ="">initial setting ?
1. A on, B on, C on.
2. A on, B on, C off.
3. A on, B off, C on.
4. A on, B off, C off.
5. A off, B on, C off.
Ans : C
5. In < name="IL_MARKER" ="">case only switch B is on in the second setting, which
among the following could have been the < name="IL_MARKER" ="">initial setting ?
1. A on, B on, C on.
2. A on, B off, C on.
3. A off, B on, C off.
4. A off, B off, C on.
5. A off, B off, C off.
Ans: C
. It was vacation time, and so I decided to visit my cousin's home. What a grand time we
had! In the mornings,
we both would go for a jog. The evenings were spent on the tennis court. Tiring as these
activities were, we could
manage only one per day, i.e., either we went for a jog or played tennis each day. There
were days when we felt
lazy and stayed home all day long. Now, there were 15 mornings when we did nothing, 9
evenings when we stayed
at home, and a total of 12 days when we jogged or played tennis. For how many days did
I stay at my cousin's
place?
A. 18 B. 14 C. 15 D. 20
Ans: A
7. If you were to construct a 7 7 checkered square (i.e., a 7 7 chess board), how many
rectangles would there be
in total? You need to include squares too because a square is a special kind of rectangle.
A. 784 B. 918 C. 842 D. 7
Ans: A
8. A block of wood in the form of a cuboid 4" 10" 13" has all its six faces painted pink.
If the wooden block is cut
into 520 cubes of 1" 1" 1", how many of these would have pink paint on them?
A. 348 B. 352 C. 344 D. 340
Ans: C
9. A tennis championship is played on a knock-out basis, i.e., a player is out of the
tournament when he loses a match.
How many players participate in the tournament if 127 matches are totally played?
A. 13 B. 144 C. 124 D. 128
Ans: C
10. A large water tank has two inlet pipes (a large one and a small one) and one outlet
pipe. It takes 3 hours to fill the
tank with the large inlet pipe. On the other hand, it takes 4 hours to fill the tank with
the small inlet pipe. The outlet
pipe allows the full tank to be emptied in 7 hours. What fraction of the tank (initially
empty) will be filled in 1.70
hours if all three pipes are in operation?
A. 0.75 B. 0.85 C. 0.80 D. 0.70
Ans: A
Answer Questions to 5 on the basis of the information given below:
K, L, M, N, P, Q, R, S, U and W are the only ten members in a department. There is a
proposal to form a team from within the members of the department, subject to the
following conditions:

A team must include exactly one among P, R, and S.
A team must include either M or Q, but not both.
If a team includes K, then it must also include L, and vice versa.
If a team includes one among S, U, and W, then it must also include the other two.
L and N cannot be members of the same team.
L and U cannot be members of the same team.

The size of a team is defined as the number of members in the team.

11. What could be the size of a team that includes K?
A. Only 2
B. 3 or 4
C. 2 or 3
D. Only 4
E. 2 or 4
Ans: D
12. In how many ways a team can be constituted so that the team includes N?
A. 5 B. C. 4 D. 2 E. 3
Ans: B
13. What would be the size of the largest possible team?
A. cannot be determined B. C. 7 D. 8 E. 5
Ans: E
14. Who can be a member of a team of size 5?
A. R B. L C. P D. M E. K
Ans: D
15 Who cannot be a member of a team of size 3?
A. P B. L C. M D. N E. Q
Ans: B
1 On a certain day it took Bill three times as long to drive from home to work as it took
Sue to drive from home to
work. How many kilometers did Bill drive from home to work?
(1) Sue drove 10 kilometers from home to work, and the ratio of distance driven from
home to work time to drive
from home to work was the same for Bill and Sue that day.
(2) The ratio of distance driven from home to work time to drive from home to work for
Sue that day was 4
kilometers per hour.
A. Both statements 1 and 2 together are sufficient to answer the question but neither
statement is sufficient alone.
B. Statement 1 alone is sufficient but statement 2 alone is not sufficient to answer the
question asked
C. Each statement alone is sufficient to answer the question.
D. Statements 1 and 2 are not sufficient to answer the question asked and additional data is
needed to answer the statements
E. Statement 2 alone is sufficient but statement 1 alone is not sufficient to answer the
question asked
Ans:B
17. If x and y are nonzero integers, is x/y an integer?
(1) x is the product of 2 and some other integer.
(2) There is only one pair of positive integers whose product equals y
A. Statement 1 alone is sufficient but statement 2 alone is not sufficient to answer the
question asked.
B. Statement 2 alone is sufficient but statement 1 alone is not sufficient to answer the
question asked.
C. Each statement alone is sufficient to answer the question.
D. Both statements 1 and 2 together are sufficient to answer the question but neither
statement is sufficient alone.
E. Statements 1 and 2 are not sufficient to answer the question asked and additional data is
needed to answer the statements.
Ans: E
18 If n is an integer between 2 and 100 and if n is also the square of an integer, what is
the value of n?
(1) n is the cube of an integer.
(2) n is even
A. Each statement alone is sufficient to answer the question.
B. Statement 2 alone is sufficient but statement 1 alone is not sufficient to answer the
question asked.
C. Statement 1 alone is sufficient but statement 2 alone is not sufficient to answer the
question asked.
D. Both statements 1 and 2 together are sufficient to answer the question but neither
statement is sufficient alone.
E. Statements 1 and 2 are not sufficient to answer the question asked and additional
data is needed to answer
the statements.
Ans: C
19. What is the average distance that automobile D travels on one full tank of gasoline?
(1) Automobile D averages 8.5 kilometers per liter of gasoline.
(2) The gasoline tank of automobile D holds exactly 40 liters of gasoline.
A. Each statement alone is sufficient to answer the question.
B. Statement 1 alone is sufficient but statement 2 alone is not sufficient to answer the
question asked.
C. Both statements 1 and 2 together are sufficient to answer the question but neither
statement is sufficient alone.
D. Statement 2 alone is sufficient but statement 1 alone is not sufficient to answer the
question asked.
E. Statements 1 and 2 are not sufficient to answer the question asked and additional
data is needed to answer the
statements.
Ans: C
20 What fraction of his salary did Mr.Johnson put into savings last week?
(1) Last week Mr. Johnson put $17 into savings.
(2) Last week Mr. Johnson put 5% of his salary into savings.
Ans: D
Directions2- 25: In each of the questions from to 5 a statement followed by two
arguments is given You have to decide which of the given statements is a strong
argument and which is weak argument ive answer (A) if only first argument is
strong, give answer (B) if only second argument is strong, give answer (C) if
either first or second argument is strong, give answer (D) if neither first nor
second argument is strong and given answer (E) if both first and second
arguments are strong

21. Statement :Should military training be made compulsory in our country like other
countries?
Arguments : I. Yes, every citizen has right to protect his country.
II. No, it is against the policy of non-violence.
Ans : (A)

22. Statement : Should octroi be abolished?
Arguments : I. No, it will adversely affect government revenues.
II. Yes, it will eliminate an important source of corruption.
Ans : (E)

23. Statement : Should competitive examinations for selecting candidates for jobs, be of
objective test only?
Arguments :I. Yes, the assessment of objective test is reliable.
II. No, the number of questions to be answered is always very large.
Ans : (A)

24. Statement : Should those who receive dowry be encouraged ?
Arguments : I. Yes, due to this, the number of those who receive dowry will decrease.
II. No, due to this the number of those who receive dowry will increase.
Ans : (B)

25. Statement : Should jobs be delinked with academic degrees and diplomas?
Arguments :I. Yes, a very large number of candidates will apply so that the
competition will be tough and the
candidates will prepare more.
II. No, importance of higher education will be diminished.
Ans : (E)
2. If '78' means-'Society Family Husbandry', '574' means-'Husbandry Health Control',
'342' means-'Health
Census shop', then-
(I) Which code has been used for 'Health' ?
(A) 7
(B) 5
(C) 4
(D) 2
(E) None of these
Ans : (C)

27. (II) Which code has been used for 'Census' ?
(A) 3 or 2
(B) 3 or 7
(C) 4
(D) 5
(E)
Ans : (A)

28. (III) For which word code '' has been used?
(A) Society
(B) Family
(C) Husbandry
(D) Society or Family
(E) None of these'
Ans : (D)

29. If 'KRN' means-'Callous collission life', 'RTP' means-'Life very sad', 'NPD' means-
'Collission sad future' then
what is the code used for 'Callous' ?
(A) R
(B) N
(C) K
(D) Can not be determined
(E) None of these
Ans : (C)

30. If 'MLT' means-'Day is clear', 'LKS' means-'Life is sad', 'SMMO' means-'Clear or sad',
then what is the
code used for 'Day' ?
(A) T
(B) K
(C) MO
(D) L
(E) None of these
Ans : (A)
3 English (30 quest)
Directions : In each of the following questions, four sentences are given Choose
the one which is grammatically correct
1. A) wrote to him yesterday
B) This is the road to go
C) Let us aim to do good
D) Take down his address in your copy
Ans: A
2. The principal and Secretary are leave
B) A number of students has failed in the examination
C) It is I who has committed this crime
D) many a man runs after money
Ans: D
3. A) These three boys hate each other
B) Sanjeev lent Rs.2000 to Vincent
C) I, you and he belong to the same village
D) Anyone of the two students can solve this sum
Ans: B
4. A)He is a man whom I know is trustworthy
B) Until you remain idle you will make no progress
C) We talked about the weather
D) Half dozen books were burnt
Ans: C
5. A)The proceeds of the charity show are for riot victims.
B) He asked Ajay and I to go
C) The weather of this place does not suit me
D) Either Rajesh or his friends has done it.
Ans: A
Directions 6-0: In each of the following questions, there is a certain relationship
between two given words on one side of : : and one word is given on another side
of : :while another word is to be found from the given alternatives, having the
same relation with this word as the words of the given pair bear Choose the
correct alternative
. Eye :Myopia : : Teeth : ?
(A) Pyorrhea (B) Cataract (C) Trachoma (D) Eczema
Ans: (A)
7. Scribble : Write : : Stammer : ?
(A) Walk (B) Play (C) Speak (D) Dance
Ans: (C)
8. Errata : Books : : flaws:?
(A) Manuscripts (B) Metals (C) Speech (D) Charter
Ans: (B)
9. Breeze : Cyclone : : Drizzle : ?
(A) earth quake (B) Storm (C) Flood (D) Down pour
Ans: (D)
10. Venerate : Worship : : Extol : ?
(A) Glorify (B) Homage (C) Compliment (D) Recommend
Ans: (A)
Directions for Questions to 5 The passage given below is followed by a set of
five questions Choose the most appropriate answer to each question
When I was little, children were bought two kinds of ice cream, sold from those white
wagons with the canopies made of silvery metal: either the two-cent cone or the four-cent
ice cream pie. The two-cent cone was very small, in fact it could fit comfortably into a
childs hand, and it was made by taking the ice cream from its container with a special
scoop and piling it on the cone. Granny always suggested I eat only a part of the cone, then
throw away the pointed end, because it had been touched by the vendors hand (though
that was the best part, nice and crunchy, and it was regularly eaten in secret, after a
pretense of discarding it).

The four-cent pie was made by a special little machine, also silvery, which pressed two disks
of sweet biscuit against a cylindrical section of ice cream. First you had to thrust your
tongue into the gap between the biscuits until it touched the central nucleus of ice cream;
then, gradually, you ate the whole thing, the biscuit surfaces softening as they became
soaked in creamy nectar. Granny had no advice to give here: in theory the pies had been
touched only by the machine; in practice, the vendor had held them against his hand while
giving them to us, but it was impossible to isolate the contaminated area.

I was fascinated, however, by some of my peers, whose parents bought them not a four-
cent pie but two two-cent cones. These privileged children advanced proudly with one cone
in their right hand and one in their left; and expertly moving their head from side to side,
they licked first one, then the other. This liturgy seemed to me so sumptuously enviable,
that many times I asked to be allowed to celebrate it. In vain. My elders were inflexible: a
four-cent ice, yes; but two two-cent ones, absolutely no.

As anyone can see, neither mathematics nor economy nor dietetics justified this refusal. Nor
did hygiene, assuming that in due course the tips of both cones were discarded. The
pathetic, and obviously mendacious, justification was that a boy concerned with turning his
eyes from one cone to the other was more inclined to stumble over stones, steps, or cracks
in the pavement. I dimly sensed that there was another secret justification, cruelly
pedagogical, but I was unable to grasp it.

Today, citizen and victim of a consumer society, a civilization of excess and waste (which
the society of the thirties was not), I realize that those dear and now departed elders were
right. Two two-cent cones instead of one at four cents did not signify squandering,
economically speaking, but symbolically they surely did. It was for this precise reason, that
I yearned for them: because two ice creams suggested excess. And this was precisely why
they were denied me: because they looked indecent, an insult to poverty, a display of
fictitious privilege, a boast of wealth. Only spoiled children ate two cones at once, those
children who in fairy tales were rightly punished, as Pinocchio was when he rejected the skin
and the stalk. And parents who encouraged this weakness, appropriate to little parvenus,
were bringing up their children in the foolish theater of "Id like to but I cant. They were
preparing them to turn up at tourist-class cheek-in with a fake Gucci bag bought from a
street peddler on the beach at Rimini.

Nowadays the moralist risks seeming at odds with morality, in a world where the consumer
civilization now wants even adults to be spoiled, and promises them always something
more, from the wristwatch in the box of detergent to the bonus bangle sheathed, with the
magazine it accompanies, in a plastic envelope. Like the parents of those ambidextrous
gluttons I so envied, the consumer civilization pretends to give more, but actually gives, for
four cents, what is worth four cents. You will throw away the old transistor radio to
purchase the new one, that boasts an alarm clock as well, but some inexplicable defect in
the mechanism will guarantee that the radio lasts only a year. The new cheap car will have
leather seats, double side mirrors adjustable from inside, and a paneled dashboard, but it
will not last nearly so long as the glorious old Fiat 500, which, even when it broke down,
could be started again with a kick.

The morality of the old days made Spartans of us all, while todays morality wants all of us
to be Sybarites.
11. Which of the following cannot be inferred from the passage?
A. Todays society is more extravagant than the society of the 1930s.
B. The act of eating two ice cream cones in akin to a ceremonial process.
C. Elders rightly suggested that a boy turning eyes from one cone to the other was more
likely to fall.
D. Despite seeming to promise more, the consumer civilization gives away exactly what the
thing is worth.
E. The consumer civilization attempts to spoil children and adults alike.

Ans: C
12. In the passage, the phrase "little parvenus refers to
A. naughty midgets
B. old hags
C. arrogant people
D. young upstarts
E. foolish kids
Ans: D
13. The author pined for two-cent cones instead of one four-cent pie because
A. it made dietetic sense
B. it suggested intemperance
C. it was more fun
D. it had a visual appeal
E. he was a glutton
Ans: B
14. What does the author mean by "now a days the moralist risks seeming at odds with
morality?
A. The moralist of yesterday have become immoral today
B. The concept of morality has changed over the years
D. The risks associated with immorality have gone up
E. The purists view of morality is fast becoming popular
Ans: B
15. According to the author, the justification for refusal to let him eat two cones was
plausibly
A. didactic
B. dietetic
C. dialectic
D. diatonic
E. diastolic
Ans: A.
Directions -5: In each of the following questions, out of the given alternatives,
choose the one which best expresses the meaning of the given word
1. Ridicule
(A) recognize (B) continuous (C) gobble (D) ludicrous
Ans: (C)
17. argantuan
(A) different (B) removable (C) colossal (D) minuscule
Ans: (C)
18. Savvy
(A) introduced (B) underestimated (C) knowledgeable (D) tremulous
Ans: (C)
19 Abandonment
A. Care B. Protection C. Concern D. Desertion
Ans: (D)
20. levied
imposed believed requested correlated
Ans: A
Directions 2 to 25: Pick out from the words, given below each sentence, the
word which would complete the sentence correctly and meaningfully
21.The pot officer told me to wait, but I..................a year now and my parcel has not
come yet.
A) waited B) am waiting C) had waited D) have waited.
Ans: A
22. A blind man is no judge.............colours.
A) of B) for C) in D) of the
Ans: C
23. He is........... Minister of the Central Government
A) a best and honest B) the best and honest C) best and the most honest D) the
best and most honest
Ans: A
24. Over indulgence.............. character as well as physical stamina.
A) strengthens B) stimulate C) debilitate D) maintains
Ans: B
25.Usually the ascent of a mountain face is much easier than the...................
A) fall B) decent C) descent D) descend
Ans: D
Directions for Questions 26-30: In each of the following questions there are
sentences that form a paragraph Identify the sentence's or part's of sentence's
that is/are correct in terms of grammar and usage (including spelling, punctuation
and logical consistency) Then, choose the most appropriate option
2.A. In 1849, a poor Bavarian immigrant named Levi Strauss
B. landed in San Francisco, California,
C. at the invitation of his brother-in-law David Stern
D. owner of dry goods business.
E. This dry goods business would later became known as Levi Strauss & Company.
A. B only
B. B and C
C. A and B
D. A only
E. A, B and D
Ans. A
27.A. In response to the allegations and condemnation pouring in,
B. Nike implemented comprehensive changes in their labour policy.
C. Perhaps sensing the rising tide of global labour concerns,
D. from the public would become a prominent media issue,
E. Nike sought to be a industry leader in employee relations.
A. D and E
B. D only
C. A and E
D. A and D
E. B, C and E
Ans: D
28.A. Charges and counter charges mean nothing
B. to the few million who have lost their home.
C. The nightmare is far from over, for the government
D. is still unable to reach hundreds who are marooned.
E. The death count have just begun.
A. A only
B. C only
C. A and C
D. A, C and D
E. D only
Ans: D
29.A. I did not know what to make of you.
B. Because youd lived in India, I associate you more with my parents than with me.
C. And yet you were unlike my cousins in Calcutta, who seem so innocent and
obedient when I visited them
D. You were not curious about me in the least.
E. Although you did make effort to meet me.
A. A only
B. A and B
C. A and E
D. D only
E. A and D
Ans : E
30.A. I was asked to stop writing
B. She denied to go with me
C. My hairs stood on end
D. I am reading this novel for four days.
E. He as well as you are guilty.
A. A only
B. A and B
C. A and E
D. D only
E. A and D
Ans : A
L&T Q & A- IV

WRITTEN TEST
Total 90 questions , 90 minutes ---SECTIONAL CUTOFF IS THERE
1. Aptitude (30 quest)
2. Reasoning (30 quest)
3. English (30 quest)
Aptitude (30 quest)
1. The average salary of 3 workers is 95 Rs. per week. If one earns Rs.115 and second
earns Rs.5 how much is the
salary of the 3rd worker.
Ans05
2. A 1 stored building has 12000 sq.feet on each floor. Company A rents 7 floors and
company B rents 4 floors.
What is the number of sq.feet of unrented floor space.
Ans60000

3.During a given week A programmer spends 1/4 of his time preparing flow chart, 3/8 of his
time coding and the
rest of the time in debugging the programs. If he works 48 hours during the week , how
many hours did he spend
debugging the program.
Ans 8
4. A company installed 3 machines at the beginning of the year. In March they installed 9
additional machines and
then disconnected 18 in August. How many were still installed at the end of the year.
Ans 2

5. A man owns 2/3 of the market research beauro business and sells 3/4 of his shares for
Rs. 75000. What is the
value of Business ?
Ans50000
. If 12 file cabinets require 18 feet of wall space, how many feet of wall space will 30
cabinets require?
Ans45

7. A computer printer produced 17,400 lines in a given day. If the printer was in operation
for seven hours during the
day, how many lines did it print per minute?
Ans420
8. From its total income, A sales company spent Rs.20,000 for advertising, half of the
remainder on commissions and
had Rs.000 left. What was its total income?
Ans32000
9. On Monday a banker processed a batch of cheques, on Tuesday she processed three
times as many, and on
Wednesday she processed 4000 cheques. In the three days, she processed 1000
cheques. How many did she
process on Tuesday?
Ans9000

10. The cost of four dozen proof machine ribbons and five dozen accouting machine ribbons
was Rs.10/-. If one
dozen accounting machine ribbons cost Rs.20/-, what is the cost of a dozen proof
machine ribbons?
AnsRs5

11. If a clerk can process 80 cheques in half an hour, how many cheques can she process in
a seven and one half
hour day?
Ans200

12. In a library, there are two racks with 40 books per rack. On a given day, 30 books were
issued. What fraction
remained in the racks?
Ans5/8

13. The average length of three tapes is 800 feet. None of the tapes is less than 400 feet.
What is the greatest
possible length of one of the other tapes?
Ans600

14. A company rented a machine for Rs.700/- a month. Five years later the treasurer
calculated that if the company
had purchased the machine and paid Rs.100/- monthly maintenance charge, the
company would have saved
Rs.2000/-. What was the purchase price of the machine?
AnsRs34000

15. Two computers each produced 48000 public utility bills in a day. One computer printed
bills at the rate of 900 an
hour and the other at the rate of 7800 an hour. When the first computer finished its
run, how many bills did the
other computer still have to print?
Ans9000

1. If a salesman's average is a new order every other week, he will break the office record
of the year. However,
after 28 weeks, he is six orders behind schedule. In what proportion of the remaining
weeks does he have to
obtain a new order to break the record?
Ans3/4
17 On a given day, a bank had 1000 cheques returned by customers. Inspection of the
first 800 cheques indicated
that 100 of those 800 had errors and were therefore the available immediately
for dataprocessing. On this basis,
how many cheques would be available immediately for data processing on that day?
Ans4000

18. A tape manufacturer reduces the price of his heavy duty tape from Rs.30/- to Rs.28/- a
reel and the price of a
regular tape from Rs.24/- to Rs.23/- a reel. A computing centre normally spends
Rs.1440/- a month for tapes
and 3/4 of this is for heavy duty tapes. How much will they save a month under the
new prices?

AnsRs8

19. The dimensions of a certain machine are 48" X 30" X 52". If the size of the machine is
increased proportionately
until the sum of its dimensions equals 15", what will be the increase in the shortest
side?
Ans 6"
20 In a certain company, 20% of the men and 40% of the women attended the annual
company picnic. If 35% of all
the employees are man, what percent of all the employees went to the picnic?
Ans33%

21. It cost a college Rs.0.70 a copy to produce a Programme for the homecoming football
game. If Rs.15,000/- was
received for advertisements in the programme, how many copies at Rs.0.50 a copy
must be sold to make a profit
of Rs.8000/- ?
Ans 35000
22. If the digits of my present age are reversed then i get the age of my son. If 1 year ago
my age was twice as that
of my son. Find my present age.
Ans father-3, son-3
23. %here ,re 6561 b,lls out oI them 1 is he,vy. Find the min. no. oI times the b,lls h,ve to be
weighed Ior Iinding out
the he,vy b,ll.
ns. 8

24.II I w,lk with 30 miles/hr i re,ch 1 hour beIore ,nd iI i w,lk with 20 miles/hr i re,ch 1 hour
l,te. Find the dist,nce
between 2 points ,nd the ex,ct time oI re,ching destin,tion is 11 ,m then Iind the speed with
which it w,lks.
ns. 120miles and 24 miles/hr

25. A thief steals half the total no of loaves of bread plus 1/2 loaf from a bakery. A second
thief steals half the
remaining no of loaves plus 1/2 loaf and so on. After the 5th thief has stolen there are
no more loaves left in the
bakery. What was the total no of loaves did the bakery have at the beginning.
Ans: 3
2. A person needs steps to cover a distance of one slab. If he increases his foot length
(step length) by 3 inches he
needs only 5 steps to cover the slabs length. What is the length of the each slab.
Ans: 3 inches
27. There are 19 red balls and one black ball. Ten balls are put in one jar and the remaining
10 are put in another jar.
What is the possibility that the black is in the right jar.
Ans: /2
28. There is one lily in the pond on 1st june . There are two in the pond on 2nd june . There
are four on 3rd june and
so on. The pond is full with lilies by the end of the june .
(i) On which date the pond is half full?
Ans: 29th
29. A lorry starts from Banglore to Mysore at .00 A.M, 7.00 A.M, 8.00 am.....10 pm.
Similarly one another starts
from Mysore to Banglore at .00 am,7.00 am, 8.00 am.....10.00pm. A lorry takes 9
hours to travel from
Banglore to Mysore and vice versa.

(i) A lorry which has started at .00 am will cross how many lorries.
Ans: 0
30. A person meets a train at a railway station coming daily at a particular time . One day
he is late by 25 minutes, and
he meets the train 5 k.m. before the station. If his speed is 12 kmph, what is the speed
of the train ?
Ans: 60 kmph
2 Reasoning (30 quest)
Answer Questions to 5 on the basis of the information given below:
The only people to attend a conference were four ship captains and the first mates of three
of those captains. The captains were L, M, N and O; the first mates were A, D and G Each
person in turn delivered a report to the assembly as follows:
Each of the first mates delivered their report exactly after his or her captain. The first
captain to speak was M, and captain N spoke after him.
1. Among the following which is not an appropriate order of delivered reports?
1. M, A, N, G, O, L, D
2. M, D, N, G, L, O, A
3. M, N, A, L, D, O, G
4. M, N, A, O, D, L, G
5. M, N, G, D, O, L, A
Ans : E
2. In case L speaks after A, and A is the third of the first mates < name="IL_MARKER"
="">to speak, then among the following statements which would be untrue?
1. O spoke immediately after G.
2. The order of the first four speakers was M, G, N, D.
3. O's first mate was present.
4. A was the fourth speaker after M.
5. < name="IL_MARKER" ="">The captains spoke in the order M, N, O, L.
Ans : D
3. Among the following statements which statement must be true?
1. In case the second speaker was a captain, the seventh speaker was a first mate.
2. In case the second speaker was a first mate, < name="IL_MARKER" ="">the
seventhspeaker was a captain.
3. In case the third speaker was a first mate, < name="IL_MARKER" ="">the
seventhspeaker was a captain.
4. In case the third speaker was a captain, < name="IL_MARKER" ="">the
seventhspeaker was a first mate.
5. In case < name="IL_MARKER" ="">the seventh speaker was a first mate, the first
and third speakers were captains.
Ans : A
4. In case A spoke immediately after L and immediately before O, and O was not the last
speaker, L spoke
1. second
2. third
3. fourth
4. fifth
5. sixth
Ans : C
5. In case G is M's first mate, D could be the person who spoke immediately
1. prior to T
2. prior to L
3. prior to V
4. after T
5. after V
Ans : D
. At the college entrance exam, a candidate is admitted according to whether he has
passed or failed the test. Of the
candidates who are really capable, 80 % pass the test and of the incapable, 25 % pass
the test. Given that 40 % of
the candidates are really capable, then the proportion of the really capable students who
can pass the test to the
total students who can pass is about:
A. 8% B. 70% C. 75% D. 73%
Ans: B
7. Excluding stoppages, the speed of a bus is 54 km/hr and including stoppages, it is 45
km/hr. For how many minutes
does the bus stop per hour ?
A. 12 B. 10 C. 9 D. 15
Ans: B
8. A boy goes to see a film and finds a man who is his relative. The man is the husband of
the sister of his mother. How
is the man related to the boy?
A. Uncle B. Brother C. Nephew D. None of the above
Ans: A
9. The ratio between the number of passengers travelling by I & II Class between two
railway stations is 1 : 50,
whereas the ratio of the I & II Class fares between the same stations is 3 : 1. If on a
particular day, Rs. 1325 were
collected from the passengers travelling between these stations, what was the amount
collected from the II Class
passengers ?
A. Rs.1250 B. Rs. 1100 C. Rs. 1000 D. Rs.1150

Ans: A
10. When Rajeev was born, his father was 32 years older than his brother and his mother
was 25 years older than
his sister. If Rajeev's brother is years older than Rajeev and his mother is 3 years
younger than his father, how
old was Rajeev's sister when Rajeev was born?
A. 15 Years B. 7 Years C. 17 Years D. 10 Years
Ans: D
11. If STRAY is coded as TUSBZ then how will MOURN be coded?
(A) LPVSO
(B) NPVSO
(C) NVPSO
(D) NPSVO
(E) None of these
Ans : (B)
12. If MAGIC is coded as PXJFF then how will LEASH be coded?
(A) PBDVD
(B) OBDVL
(C) OHEOK
(D) OBDPK
(E) None of these
Ans : (D)
13. If DOWN is coded as FQYP then how will WITH be coded?
(A) KYN
(B) JYK
(C) YKVJ
(D) JKVY
(E) None of these
Ans : (C)
14. If BINARY is coded as DHPZTK then how will KIDNAP be coded?
(A) MKFPCQ
(B) MHFPZQ
(C) IKFMYO
(D) MHFMCO
(E) None of these
Ans : (D)
15. If RASCAL is coded as QZRBZK then how will SOLDER be coded?
(A) RNMEDQ
(B) RPKEDS
(C) TPMEFS
(D) RNKCDQ
(E) None of these
Ans : (D)
Directions 6:20 : In each question below are given two statements followed by
two conclusions numbered I and II You have to take the given two statements
to be true even if they seem to be at variance from commonly known facts Read
the conclusions and then decide which of the given conclusions logically follows
from the two given statements, disregarding commonly known facts
ive answer (A) if only conclusion I follows; (B) if only conclusion II follows; (C) if
either I or II follows (D) if neither I or II follows and (E) if both I and II
1.Statements : All puppets are dolls
All dolls are toys
Conclusions : I. Some toys are puppets
II. All toys are puppets
Ans: A
17.Statements : All apples are oranges
Some oranges are papayas
Conclusions : I. Some apples are papayas
II. Some papayas are apples
Ans: D
18. Statements :Some players are singers
All singers are tall
Conclusions : I. Some players are tall
II. All players are tall
Ans: A
19.Statements : All coins are crows
Some crows are pens
Conclusions : I. No pen is coin
II. Some coins are pens
Ans: D
20.Statements : All men are married
Some men are educated
Conclusions : I. Some married are educated
II. Some educated are married
Ans: E
21. Pointing to a man Snehlata says, "He is the only son of my father's father." How is
Snehlata related to the man ?
(A) Mother
(B) Grand daughter
(C) Niece
(D) Sister
(E) None of these
Ans : (E)
22. Pointing to a lady in photograph, Madhurendra said, "Her mother is the only daughter of
my mother's mother."
How is Madhurendra related to the lady?
(A) Nephew
(B) Uncle
(C) Maternal uncle
(D) Brother
(E) None of these
Ans : (D)
23. If P x Q means- 'P is the brother of Q', P Q means 'P is the son of Q' and 'P - Q'
means 'P is the sister of Q' ,
then which of the following relations will show that Q is the maternal uncle of P ?
(A) Q x R P
(B) Q R - P
(C) P x R - Q
(D) P R - Q
(E) None of these
Ans : (D)
24. In the class of 40 students, if Sanju is at 30th place from one end, what is his position
from the other end?
(A) 9th
(B) 12th
(C) 10th
(D) 11th
(E) None of these
Ans : (D)
25. In a row of girls, Nivedita is 15th from the left and Vimla is 23rd from the right. If they
interchange their positions,
then Nivedita becomes 18 th from the left. Then at what position will Vimla be from the
right?
(A) 25th
(B) 24th
(C) 2th
(D) 20th
(E) None of these
Ans : (C)
2 As 'Hindu worshiper' is related to 'Temple' in the same way 'Maulvi' is related to what?
(A) Monastery
(B) Church
(C) Mosque
(D) Sikh temple
(E) None of these
Ans : (C)
27. As 'Hungry' is related to 'Food' in the same way 'Thirsty' is related to what?
(A) Drink
(B) Tea
(C) Coffee
(D) Juice
(E) Waler
Ans : (E)
28. As 'Fly' is related to 'Parrot' in the same way 'Creep' is related to what?
(A) Snake
(B) Rabbit
(C) Fish
(D) Crocodile
(E) Sparrow
Ans : (A)
29. If `M denotes `, `K, denotes `-, `T denotes ` and `R denotes `+, then; 20 K 1 T
8 M 4 R = ?
(A) 18
(B) -
(C) - 12
(D) - 18
(E) None of these
Ans: B
30. How many such pairs of letters are there in the word TERMINATE each of which has as
many letters between them in the word as in the English alphabet ?
(A) None
(B) One
(C) Two
(D) Three
(E) More than three
Ans: B
3 English (30 quest)
Directions -5: In each of the following questions, there is a certain relationship
between two given words on one side of : : and one word is given on another side
of : :while another word is to be found from the given alternatives, having the
same relation with this word as the words of the given pair bear Choose the
correct alternative
1.Oxygen : Burn : : Carbon dioxide : ?
(A) Isolate (B) Foam (C) Extinguish (D) Explode
Ans: (C)
2. Grain : Stock : : Stick : ?
(A) Heap (B) Bundle (C) Collection (D) String
Ans: (B)
3. Planet : Orbit : : Projectile : ?
(A) Trajectory (B) Track (C) Milky way (D) Path
Ans: (A)
4. Genuine : Authentic : : Mirage : ?
(A) Image (B) Transpiration (C) Reflection (D) Illusion
Ans: (D)
5.Illiteracy : Education : : Flood : ?
(A) Rain (B) Bridge (C) Dam (D) River
Ans: (C)
Directions for Questions 6 to0 :Each of the following sentences has been divided
into four parts There is an error in one of the parts Point out the part which has
an error
.A) I was astonished by the highly exciting tricks /
B) the acrobat displayed on the rope /
C) at great risk of like
D) at such an advanced age.
Ans: A
7. A) We, who are fortunate enough /
B) to have lived in the present century, /
C) hardly realize how our ancestors suffered /
D) from the belief in the existence of evil spirits.
Ans: B
8. A) We were expecting at least twenty delegates /
B) to participate in the seminar, /
C) but when I reached the hall,
D) I found no any delegate present there.
Ans: D
9. A) We bought five dozens pencils from a shop,/
B) but on opening in the packet at home /
C) we found five pencils short, /
D) but the shopkeeper did not accept the responsibility of the shortage.
Ans: A
10. A) I wanted to book a parcel weighting twenty and a half kilos /
B) but the booking -clerk refused to book it /
C) on the ground
D) that it was not properly sealed.
Ans: A
Directions for Questions -5: In each of the following questions, a paragraph or
a sentence has been broken up into different parts The parts have been scrambled
and numbered as given below Choose the correct order of these parts from the
given alternatives
11. A. The potential exchanges between the officials of IBBF and the Maharashtra Body-
Building Association has all the trappings of a drama we are accustomed to.
B. In the case of sportspersons, there is room for some sympathy, but the apathy of
the administrators, which has even led to sanctions from international bodies, is
unpardonable.
C. A case in the point is the hefty penalty of US $10,000 slapped on the Indian Body-
Building Federation for not fulfilling its commitment for holding the Asian
Championships in Mumbai in October.
D. It is a matter of deep regret and concern that the sports administrators often
cause more harm to the image of the country than sportsmen and sportswomen do
through their dismal performances.

1] CABD 2] DBCA 3] DABC 4] CDBA

Ans: 2
12. A. Its cargo consisted of 38 sacks of spices and Magellan himself had been hacked to
pieces on the beach of Mactan in the Phillipines
B. So contrary to popular beliefe it was the crew of the Victoria who were the first
men to have sailed around the globe
C. In spetembre 1522 Victoria , the sole survivor of the Armada, limped into the
spanish port San Lucar , manned by a skeleton crew of 15, so weak they could not
talk
D. In septembre 1519 the Armada de Molucca of five ships and 250 sailors has set
out from San lucar de Barrameda under the command of Fernando de Magellan
E. It was to sail to the spice islands of the Malayan Archipelago where they were to
excahnge an assortemnt of bells , mirrors , and scissors for cinnamon and cloves.

1] DECAB 2] AEDCB 3] CDEAB 4] DEABC

Ans:
13. A. What came out was very large garland made out of currency notes.
B. The unsuspecting governor opened the box in full view of the gathering
C. When the RBI governor came to inaugrate the new printing press , the local unit
of the BJP handed him a gift wrapped box
D. There was a twist - the notes were all as tattered as notes could get

1] DACB 2] CABD 3] CBAD 4] DCAB

Ans:3
14. A. But in the industrial era destroying the enemys productive capacity means
bombing the factories which are located in the cities.
B. So in the agrarian era, if you need to destroy the enemys productive capacity,
what you want to do is bum his fields, or if youre really vicious, salt them.
C. Now in the information era, destroying the enemys productive capacity means
destroying the information infrastructure.
D. How do you do battle with your enemy?
E. The idea is to destroy the enemys productive capacity, and depending upon the
economic foundation, that productive capacity is different in each case.
F. With regard to defence, the purpose of the military is to defend the nation and be
prepared to do battle with its enemy.

1] FDEBAC 2] FCABED 3] DEBACF 4] DFEBAC

Ans:
15. A. The situations in which violence occurs and the nature of that violence tends to be
clearly defined at least in theory, as in the proverbial Irishmans question: `Is this a
private fight or can anyone join in?
B. So the actual risk to outsiders, though no doubt higher than our societies, is
calculable.
C. Probably the only uncontrolled applications of force are those of social superiors to
social inferiors and even here there are probably some rules.
D. However binding the obligation to kill, members of feuding families engaged in
mutual massacre will be genuinely appalled if by some mischance a bystander or
outsider is killed.

1] DABC 2] ACDB 3] CBAD 4] DBAC

Ans:
Directions for Questions 6 to 20: The passage given below is followed by a set of five
questions. Choose the most appropriate answer to each question.
To summarize the Classic Maya collapse, we can tentatively identify five strands. I
acknowledge, however, that Maya archaeologists still disagree vigorously among
themselves-in part, because the different strands evidently varied in importance among
different parts of the Maya realm; because detailed archaeological studies are available for
only some Maya sites; and because it remains puzzling why most of the Maya heartland
remained nearly empty of population and failed to recover after the collapse and after re-
growth of forests.

With those caveats, it appears to me that one strand consisted of population growth
outstripping available resources: a dilemma similar to the one foreseen by Thomas Malthus
in 1798 and being played out today in Rwanda (Chapter 10), Haiti (Chapter 11), and
elsewhere. As the archaeologist David Webster succinctly puts it, "Too many farmers grew
too many crops on too much of the landscape. Compounding that mismatch between
population and resources was the second strand: the effects of deforestation and hillside
erosion, which caused a decrease in the amount of useable farmland at a time when more
rather than less farmland was needed, and possibly exacerbated by an anthropogenic
drought resulting from deforestation, by soil nutrient depletion and other soil problems, and
by the struggle to prevent bracken ferns from overrunning the fields.

The third strand consisted of increased fighting, as more and more people fought over fewer
resources. Maya warfare, already endemic, peaked just before the collapse. That is not
surprising when one reflects that at least 5,000,000 people, perhaps many more, were
crammed into an area smaller than the state of Colorado (104,000 square miles). That
warfare would have decreased further the amount of land available for agriculture, by
creating no-mans lands between principalities where it was now unsafe to farm. Bringing
matters to a head was the strand of climate change. The drought at the time of the Classic
collapse was not the first drought that the Maya had lived through, but it was the most
severe. At the time of previous droughts, there were still uninhabited parts of the Maya
landscape, and people at a site affected by drought could save themselves by moving to
another site. However, by the time of the Classic collapse the landscape was now full, there
was no useful unoccupied land in the vicinity on which to begin anew, and the whole
population could not be accommodated in the few areas that continued to have reliable
water supplies.

As our fifth strand, we have to wonder why the kings and nobles failed to recognize and
solve these seemingly obvious problems undermining their society. Their attention was
evidently focused on their short-term concerns of enriching themselves, waging wars,
erecting monuments, competing with each other, and extracting enough food from the
peasants to support all those activities. Like most leaders throughout human history, the
Maya kings and nobles did not heed long-term problems, insofar as they perceived them.
We shall return to this theme in Chapter 14.

Finally, while we still have some other past societies to consider in this book before we
switch our attention to the modern world, we must already be struck by some parallels
between the Maya and the past societies discussed in Chapters 2-4. As on Easter Island,
Mangareva, and among the Anasazi, Maya environmental and population problems led to
increasing warfare and civil strife. As on Easter Island and at Chaco Canyon, Maya peak
population numbers were followed swiftly by political and social collapse. Paralleling the
eventual extension of agriculture from Easter Islands coastal lowlands to its uplands, and
from the Mimbres floodplain to the hills, Copans inhabitants also expanded from the
floodplain to the more fragile hill slopes, leaving them with a larger population to feed when
the agricultural boom in the hills went bust. Like Easter Island chiefs erecting ever larger
statues, eventually crowned by pukao, and like Anasazi elite treating themselves to
necklaces of 2,000 turquoise beads, Maya kings sought to outdo each other with more and
more impressive temples, covered with thicker and thicker plaster- reminiscent in turn of
the extravagant conspicuous consumption by modern American CEOs. The passivity of
Easter chiefs and Maya kings in the face of the real big threats to their societies completes
our list of disquieting parallels.
1. According to the passage, which of the following best represents the factor that has
been cited by the author in the context of Rwanda and Haiti?
(1) Various ethnic groups competing for land and other resources.
(2) Various ethnic groups competing for limited land resources.
(3) Various ethnic groups fighting wit each other.
(4) Various ethnic groups competing for political power.
(5) Various ethnic groups fighting for their identity.
Ans: 2
17. By an anthropogenic drought, the author means
(1) A drought caused by lack of rains.
(2) A drought caused due to deforestation.
(3) A drought caused by failure to prevent bracken ferns from overrunning the fields.
(4) A drought caused by actions of human beings.
(5) A drought caused by climate changes.

Ans: 4
18. According to the passage, the drought at the time of Maya collapse had a different
impact compared to the droughts earlier because
(1) The Maya kings continue to be extravagant when common people were suffering.
(2) It happened at the time of collapse of leadership among Mayas.
(3) It happened when the Maya population had occupied all available land suited for
agriculture.
(4) AIt was followed by internecine warfare among Mayans.
(5) Irreversible environmental degradation led to this drought.
Ans: 3
19. According to the author, why is it difficult to explain the reasons for Maya collapse?
(1) Copan inhabitants destroyed all records of that period.
(2) The constant deforestation and hillside erosion have wiped out all traces of the Maya
kingdom.
(3) Archaeological sites of Mayas do not provide any consistent evidence.
(4) It has not been possible to ascertain which of the factors best explains as to why the
Maya civilization collapsed.
(5) At least five million people were crammed into a small area.
Ans: 4
20. Which factor has not been cited as one of the factors causing the collapse of Maya
society?
(1) Environmental degradation due to excess population.
(2) Social collapse due to excess population.
(3) Increased warfare among Maya people.
(4) Climate change.
(5) Obsession of Maya population with their own short-term concerns.
Ans: 3
Directions 2-25: Pick out from the words, given below each sentence, the word
which would complete the sentence correctly and meaningfully
21. Unhygienic circumstances..................health problems.
A) give rise to B) bring in to existence C) call for D) replace
Ans: A
22. If she were chosen for the job, she..........a good secretary.
A) will make B) can make C) would make D) would have made
Ans: C
23. Didn't you go to the restaurant yesterday...........?
A) No I did'nt B) Yes I didn't C) No, I did D) will, yes.
Ans: A
24. The Chernobil tragedy had taken place about four years.............
A) past B) ago C) before D) since
Ans: B
25.Every government servant is finding it necessary to supplement his............... by doing
something extra.
A) earnings B) profits C) remuneration D) income
Ans: D
Directions -5: In each of the following questions, out of the given alternatives,
choose the one which best expresses the meaning of the given word
2 Abolish
1. < ="FuncBtnOut(this)" style="display: inline;" id="Q_0_0_Btn"
up="FuncBtnOut(this)" ="Func" ="FuncBtnOver(this)" ="FuncBtnOver(this)"
="FuncBtnOut(this)" down="FuncBtnDown(this)"
="CheckMCAnswer(0,0,this)"> end
2. minimize
3. < ="FuncBtnOut(this)" style="display: inline;" id="Q_0_2_Btn"
up="FuncBtnOut(this)" ="Func" ="FuncBtnOver(this)" ="FuncBtnOver(this)"
="FuncBtnOut(this)" down="FuncBtnDown(this)"
="CheckMCAnswer(0,2,this)"> prevent
4. < ="FuncBtnOut(this)" id="Q_0_3_Btn" up="FuncBtnOut(this)" ="Func"
="FuncBtnOver(this)" ="FuncBtnOver(this)" ="FuncBtnOut(this)"
down="FuncBtnDown(this)" ="CheckMCAnswer(0,3,this)"> denounce
Ans: A
27. make up
1. invent
2. < ="FuncBtnOut(this)" style="display: inline;" id="Q_2_1_Btn"
up="FuncBtnOut(this)" ="Func" ="FuncBtnOver(this)" ="FuncBtnOver(this)"
="FuncBtnOut(this)" down="FuncBtnDown(this)"
="CheckMCAnswer(2,1,this)">prepare
3. < ="FuncBtnOut(this)" id="Q_2_2_Btn" up="FuncBtnOut(this)" ="Func"
="FuncBtnOver(this)" ="FuncBtnOver(this)" ="FuncBtnOut(this)"
down="FuncBtnDown(this)" ="CheckMCAnswer(2,2,this)">replace
4. complete
Ans: D
28. dispatched
1. taken by force
2. ordered to go
3. < ="FuncBtnOut(this)" style="display: inline;" id="Q_3_2_Btn"
up="FuncBtnOut(this)" ="Func" ="FuncBtnOver(this)" ="FuncBtnOver(this)"
="FuncBtnOut(this)" down="FuncBtnDown(this)"
="CheckMCAnswer(3,2,this)">expelled
4. sent
Ans: D
29. ARDUOUS
1. Short
2. Difficult
3. Easy
4. Expensive
Ans: B
30. ABODE
1. Family
2. Farm
3. Car
4. Home

L&T Q & A -v

WRITTEN TEST
Total 90 questions , 90 minutes ---SECTIONAL CUTOFF IS THERE
1. Aptitude (30 quest)
2. Reasoning (30 quest)
3. English (30 quest)
Aptitude (30 quest)
1. If the thrice of three consecutive odd no is equal to the more three of twice the last no.
then find the 3rd
(largest odd no).?
Ans:
2. Rahul 12th from right and 4th from left . How many men should be added in the queue to
make a group of 28?
Ans 3

3. Present population of town is 35,000 having males and females. If The population of
males is increased by %
and if the population of females is increased by 4%, then after 1 year the population
becomes 3,700.
Find the number males and females ?
Ans8,000
4.Gets 90% of B. B gets 25% more than C. C gets 80% of D. Marks of A are given then find
the percentage marks
obtained by D.
Ans = 80%

5. Length of rectangular plot is 3 times its breadth having area 3 hectare . A man walks on
its perimeter at 4 km/hr .
Find the time required ?
Ans 2 min
Speed of trains A and B are 29 & 5 km/hr travelling in the same direction. Man in slower
train passes in 1 min.
Find the length of the train.?
Ans 20m
7 Y catches 5 times more fishes than X. If total number of fishes caught by X and Y is 48,
then number of fishes
caught by X?
Ans: 8

8 The girl's age is twice that of boy, if the boy is four years old. After four years the age of
the girl is ?
Ans: 2 years
9 A clock is late by 1 minute 27 seconds in a month. Then how much will it be late in 1
day?
Ans: 29 seconds
10 At 20% discount, a cycle is sold at a selling price of 2500 Rs. What is the actual price?
Ans: Rs 325
11. A man purchased stamps of rupees 1 and seven stamps of 50 paise. he paid Rs. 12.
how much change
he got back ?
Ans 250 Rs
12. 8 man work for days to complete a work. How many men are required to complete
same work in 1/2 day.
Ans 96 men
13. A man drives at a speed of 40 miles/hr. His wife left 30 mins. late with 50 miles/hr
speed. when will they meet ?
Ans 2 hours
14. 1,40,00,000 pencils are put up straight. all the pencils are of length range 3 to inches.
80% of the pencils have
average of five inches. so the find out the total length spanned by the pencils.
Ans 000 to 500 miles

15. One person says `his only daughter is my sons mother , what is the
relationshipbetween 1st &2nd person.
Ans :son -in -law

1 Alphabet `A to `Z. If we arrange in reverse order (Z to A) then which letter will be in
the 5th position towards
right ,taken from 11th better from left.
Ans: k

17. In a railway station, there are two trains going. One in the harbor line and one in the
main line, each having a
frequency of 10 minutes. The main line service starts at 5 o'clock and the harbor line
starts at 5.02A.M. A
man goes to the station every day to catch the first train that comes. What is the
probability of the man catching
the first train?
Ans: 08

18. Light glows for every 13 seconds . How many times did it glow between 1:57:58 and
3:20:47 am.
Ans : 383 + = 384

19 From a vessel, 1/3rd of the liquid evaporates on the first day. On the second day 3/4th
of the remaining liquid
evaporates. What fraction of the volume is present at the end of the second day.
Ans: 50%

20. There are 20 poles with a constant distance between each pole. A car takes 24 second
to reach the 12th pole.
How much will it take to reach the last pole.
Ans: 445 seconds
21. Two trains starting at same time, one from Bangalore to Mysore and other in opposite
direction arrive at their
destination 1 hr and 4 hours respectively after passing each other. How much faster is
one train from other?
Ans: Twice
22. Two trains start from stations A and B spaced 50kms apart at the same time and speed.
As the trains start,
a bird flies from one train towards the other and on reaching the second train, it flies
back to the first train. This is
repeated till the trains collide. If the speed of the trains is 25 km/h and that of the bird
is 100km/h. How much did
the bird travel till the collision.
Ans: 00kms
23. Sometime after 10:00 PM a murder took place. A witness claimed that the clock must
have stopped at the time of
the shooting. It was later found that the position of both the hands were the same but
their positions had
interchanged Tell the time of the shooting (both actual and claimed).
Ans: Time of shooting = :54 PM Claimed Time = 0:59 PM

24. 500 men are arranged in an array of 10 rows and 50 columns according to their heights.
Tallest among each row
of all are asked to fall out. And the shortest among them is A. Similarly after resuming
that to their original
positions that the shortest among each column are asked to fall out. And the tallest
among them is B. Now who is
taller among A and B ?
Ans A
25. A family X went for a vacation. Unfortunately it rained for 13 days when they were
there. But whenever it rained
in the mornings, they had clear afternoons and vice versa. In all they enjoyed 11
mornings and 12 afternoons.
How many days did they stay there totally?
Ans: 8
2. The minute and the hour hand of a watch meet every 5 minutes. How much does the
watch lose or gain time and
by how much?
Ans: ains; 5/ minutes

27. Every day a cyclist meets a train at a particular crossing. The road is straight before the
crossing and both are
traveling in the same direction. The cyclist travels with a speed of 10kmph. One day the
cyclist comes late by 25
min. and meets the train 5km before the crossing. What is the speed of the train?
Ans: 60kmph
28. A ship went on a voyage. After it had traveled 180 miles a plane started with 10 times
the speed of the ship.
Find the distance when they meet from starting point.
Ans: 200miles
29. Father's age is three years more than three times the son's age. After three years,
father's age will be ten years
more than twice the son's age. What is the father's present age?
Ans: 33 years
30. A man collects cigarette stubs and makes one full cigarette with every 8 stubs. If he
gets 4 stubs how many full
cigarettes can he smoke ?
Ans: 8+=9


2 Reasoning (30 quest)
Answer Questions to 5 on the basis of the information given below:

K, L, M, N, P, Q, R, S, U and W are the only ten members in a department. There is a
proposal to form a team from within the members of the department, subject to the
following conditions:

A team must include exactly one among P, R, and S.
A team must include either M or Q, but not both.
If a team includes K, then it must also include L, and vice versa.
If a team includes one among S, U, and W, then it must also include the other two.
L and N cannot be members of the same team.
L and U cannot be members of the same team.

The size of a team is defined as the number of members in the team.

What could be the size of a team that includes K?
A. Only 2 B. 3 or 4 C. 2 or 3 D. Only 4 E. 2 or 4
Ans: D
2. In how many ways a team can be constituted so that the team includes N?
A. 5 B. C. 4 D. 2 E. 3
Ans: B
3. What would be the size of the largest possible team?
A. cannot be determined B. C. 7 D. 8 E. 5
Ans: E
4. Who can be a member of a team of size 5?
A. R B. L C. P D. M E. K
Ans: D
5. Who cannot be a member of a team of size 3?
A. P B. L C. M D. N E. Q
Ans: B
. In an island three persons are there Jam ,Dam and Sam . sam having pencil, dam having
cap and Jam having book .
we have find who is author among them if.
(1)Sam says ,Jam is author but author wearing cap.
(2)Dam says ,I am author ,but author not having cap.
(3)Sam says, I am author ,but author having note book.
Find who is author?

Ans : jam
7. My Dad has a miniature Pyramid of Egypt. It is 3 inches in height. Dad was invited to
display it at an exhibition.
Dad felt it was too small and decided to build a scaled-up model of the Pyramid out of
material whose density is
(1 / 7) times the density of the material used for the miniature. He did a "back-of-the-
envelope" calculation to check
whether the model would be big enough. If the mass (or weight) of the miniature and the
scaled-up model are to be
the same, how many inches in height will be the scaled-up Pyramid?
A. 5.78 B.5.82 C. 5.8 D. 5.74
Ans D
8. Neelam rides her bicycle from her house at A to her club at C, via B taking the shortest
path. Then the number of
possible shortest paths that she can choose is

1. 1170
2. 30
3. 792
4. 1200
5. 93
Ans: A
9. In a triangle ABC, the lengths of the sides AB and AC equal 17.5 cm and 9 cm
respectively. Let D be a point on
the line segment BC such that AD is perpendicular to BC. If AD=3 cm, then what is the
radius (in cm) of the circle
circumscribing the triangle ABC?
1. 17.05
2. 27.85
3. 22.45
4. 32.25
5. 2.25
Ans: E
10. Gary wants to dry his clothes. It takes him 0 minutes to dry 10 clothes under the sun.
The very next day, he has
20 clothes (double the numbers) to dry the clothes. What should be the expected time
to dry double the number
of clothes?
Ans: 60 minutes
Answer Questions to 5 on the basis of the information given below:
There are women Shalini, Divya, Ritu, Rashmi, Nisha and Renu in a family of twelve
members. There are few married couples in the family and none of the grandchildren are
married. Sunil is married in to the family. Rohan, Mahesh and Jatin have a nephew Dipesh
who is the only son of Rashmi. Ravi is the paternal grandfather of Nisha. Ritu is the
daughter-in-law of Shalini. Renu is the first cousin of Dipesh. Shalini has only three
Grandchildren. Mahesh has two borthers and only one sister Rashmi and a sister-in-law
Divya. Dipeshs only unmarried maternal uncle, Jatin is the brother-in-law of Sunil. Rohan is
the paternal uncle of Nisha. Ritu has two daughters one of whom is Nisha.

11. Nisha is:
A. none of these B. Rohan's daughter C. Renu's cousin D. Jatin's mother
Ans: A

12. Dipesh is:
A. Ravi's grandson B. Sunil's nephew C. Mahesh's son D. Rohan's son
Ans: A

13. How many married couples are there in the second generation?
A. 1 B. 3 C. 4 D. 2
Ans: B
14 Which one of the following is true?
A. Ravi has only Two Married children. B. None of these.
C. Dipesh is Mahesh's son D. Ravi is the paternal grandfather of Renu
Ans: D

15. Which one of these is a married couple?
A. Rohand and Ritu B. Mahesh and Ritu C. Renu and Sunit D. Shalini and
Mahesh
Ans: B
1. If KEDGY is coded as EKDYG then how will LIGHT be coded ?
(A) ILHTG
(B) ILGHT
(C) ILGTH
(D) THGIL
(E) None of these
Ans : (C)

17. If RAVE is coded as SXWB then how will SCAW be coded ?
(A) TDBO
(B) TZBK
(C) PZXK
(D) TVXK
(E) None of these
Ans : (B)

18. If SPANK is coded as PSNAK then how will THROW be coded?
(A) HTORW
(B) HTWOR
(C) HTWRO
(D) HTRWO
(E) None of these
Ans : (A)

19. If UDOMETER is coded as DUMOTERE then how will SUBLEASE be coded?
(A) USLBESAE
(B) USLBAEES
(C) USBAELES
(D) USLBEAES
(E) None of these
Ans : (B)

20. If PURSER is coded as UPSRRE then how will PERIODIC be coded?
(A) EPRIDOIC
(B) PEIRDOCI
(C) EPIRDOCI
(D) EPIRODCI
(E) None of these
Ans : (C)

21 In a row of ladies Manorama is 20th from the right and Kanta is 10th from the left. When
they interchange their
positions Manorama becomes 25th from the right. What is the total number of ladies in
the row ?
(A) 35
(B) 34
(C) 44
(D) 24
(E) None of these
Ans : (B)

22. A number of people are standing in a row in which Kailash is 20th from the left and
Hemant is 25th from the right.
If they interchange their positions then Kailash becomes 25th from the right. How many
people are there in the
row?
(A) 49
(B) 44
(C) 45
(D) Data inadequate
(E) None of these
Ans : (D)
23. In a row of boys, Udai is 23rd from the left and Ashok is 8th from the right. When they
interchange their positions Udai becomes 18th from the right. What will be the position of
Ashok from the left ?
(A) 15th
(B) 10th
(C) 40th
(D) Data inadequate
(E) None of these
Ans : (D)
24 A lady pointing to a woman in a photograph says, "She is the only daughter of my
father-in-law." How is the woman related to the lady?
(A) Mother
(B) Sister
(C) Friend
(D) Aunt
(E) None of these
Ans : (E)

25. Pointing to a woman in a photograph a man says, "Her mother's mother is the mother
of my father." How is the man related to the woman in a photograph?
(A) Uncle
(B) Maternal cousin
(C) Nephew
(D) Grand son
(E) None of these
Ans : (B)
Directions 26-30 : In each question below are given two statements followed by
two conclusions numbered I and II You have to take the given two statements to
be true even if they seem to be at variance from commonly known facts Read the
conclusions and then decide which of the given conclusions logically follows from
the two given statements, disregarding commonly known facts
ive answer (A) if only conclusion I follows; (B) if only conclusion II follows; (C) if
either I or II follows (D) if neither I or II follows and (E) if both I and II
2. Statements : All planets are moons.
All moons are stars.
Conclusions : I. All moons are planets
II. All planets are stars
Ans: B

27. Statements : All men are dogs.
All dogs are cats.
Conclusions : I. All men are cats
II. All cats are men
Ans: A
28. Statements : All tubes are handles
All cups are handles
Conclusions : I. All cups are tubes
II. Some handles are not cups
Ans: D
29.Statements : All bags are cakes
All lamps are cakes
Conclusions : I. Some lamps are bags
II. No lamp is bag.
Ans: D
30.Statements : All flowers are stems
All stems are roots
Conclusions : I. All roots are flowers
II. All stems are flowers
Ans: D
3 English (30 quest)
Directions for Questions to5: The passage given below is followed by a set of five
questions Choose the most appropriate answer to each question
Language is not a cultural artifact that we learn the way we learn to tell time or how the
federal government works. Instead, it is a distinct piece of the biological makeup of our
brains. Language is a complex, specialized skill, which develops in the child spontaneously,
without conscious effort or formal instruction, is deployed without awareness of its
underlying logic, is qualitatively the same in every individual, and is distinct from more
general abilities to process information or behave intelligently. For these reasons some
cognitive scientists have described language as a psychological faculty, a mental organ, a
neural system, and a computational module. But I prefer the admittedly quaint term
"instinct. It conveys the idea that people know how to talk in more or less the sense that
spiders know how to spin webs. Web-spinning was not invented by some unsung spider
genius and does not depend on having had the right education or on having an aptitude for
architecture or the construction trades. Rather, spiders spin spider webs because they have
spider brains, which give them the urge to spin and the competence to succeed. Although
there are differences between webs and words, I will encourage you to see language in this
way, for it helps to make sense of the phenomena we will explore.

Thinking of language as an instinct inverts the popular wisdom, especially as it has been
passed down in the canon of the humanities and social sciences. Language is no more a
cultural invention than is upright posture. It is not a manifestation of a general capacity to
use - symbols: a three year old, we shall see, is a grammatical genius, but is quite
incompetent at the visual arts, religious iconography, traffic signs, and the other staples of
the semiotics curriculum. Though language is a magnificent ability unique to Homo sapiens
among living species, it does not call for sequestering the study of humans from the domain
of biology, for a magnificent ability unique to a particular living species is far from unique in
the animal kingdom. Some kinds of bats home in on flying insects using Doppler sonar.
Some kinds of migratory birds navigate thousands of miles by calibrating the positions of
the constellations against the time of day and year. In natures talent show we are simply a
species of primate with our own act, a knack for communicating information about who did
what to whom by modulating the sounds we make when we exhale.

Once you begin to look at language not as the ineffable essence of human uniqueness but
as a biological adoption to communicate information, it is no longer as tempting to see
language as an insidious shaper of thought, and, we shall see, it is not Moreover, seeing
language as one of natures engineering marvels - an organ with "that perfection of
structure and co-adaption which justly excites our admiration, in Darwins words- give us a
new respect for your ordinary Joe and the much-maligned English language (or any
language) The complexity of language, from the scientists point of view, is part of our
biological birth right; it is not something that parents teach their children or something that
must be elaborated in school - as Oscar Wilde said, Education is an admirable thing, but it
is well to remember from time to time that nothing that is worth knowing can be taught
"pre scholar `stacit knowledge of grammar is more sophisticated than the thickest style
manual or the most state-of-the-art computer language system, and the same applies to all
healthy human beings, even the notorious syntax-fracturing professional athlete and the,
you know, like, inarticulate teenage skateboarder. Finally, since language is the product of a
well-engineered biological instinct, we shall see that it is not nutty barrel of monkeys that
entertainer-columnists make it out to be.
1. According to the passage, which of the following does not stem from popular wisdom on
language?
A. Language is a cultural artifact.
B. Language is a cultural invention.
C. Language is learnt as we grow.
D. Language is unique to Homo sapiens.
E. Language is a psychological faculty.
Ans: E

2. Which of the following can be used to replace the "spiders know how to spin webs
analogy as used by the author?
A. A kitten learning to jump over a wall
B. Bees collecting nectar.
C. A donkey carrying a load.
D. A horse running a Derby.
E A pet dog protecting its owners property.
Ans: B
3. According to the passage, which of the following is unique to human beings?
A. Ability to use symbols while communicating with one another.
B. Ability to communicate with each other through voice modulation.
C. Ability to communicate information to other members of the species.
D. Ability to use sound as means of communication.
E. All of the above.
Ans: B
4. According to the passage, complexity of language cannot be taught by parents or at
school to children because
A. children instinctively know language.
B. children learn the language on their own.
C. language is not amenable to teaching.
D. children know language better than their teachers or parents.
E. children are born with the knowledge of semiotics.
Ans: A
5. Which of the following best summarizes the passage?
A. Language is unique to Homo sapiens.
B. Language is neither learnt nor taught.
C. Language is not a cultural invention or artifacts as it is made out.
D. Language is instinctive ability of human beings.
E. Language is use of symbols unique to human beings.
Ans: D
Directions 6-0 : In each of the following questions, there is a certain relationship
between two given words on one side of : : and one word is given on another side
of : :while another word is to be found from the given alternatives, having the
same relation with this word as the words of the given pair bear Choose the
correct alternative
. Forecast : Future : : Regret :?
(A) Present (B) Atone (C) Past (D) Sins
Ans: (C)
7. Fear : Threat : : Anger : ?
(A) Compulsion (B) Panic (C) Provocation (D) Force
Ans: (C)
9. Sculptor : Statue : : Poet : ?
(A) Canvas (B) Pen (C) Verse (D) Chisel
Ans: (C)
10. Paw : Cat : : Hoof : ?
(A) Horse (B) Lion (C) Lamb (D) Elephant
Ans: (A)
Directions -5: In each of the following questions, out of the given alternatives,
choose the one which best expresses the meaning of the given word

11. Difficult
(A) retractable (B) awe-inspiring (C) lustrous (D) formidable
Ans: (A)
12. Understatement
(A) distortion (B) estimation (C) animation (D) pretentiousness
Ans: (A)
13. Outlandish
(A) foreshadowing (B) inclusive (C) remote (D) omnipresent
Ans: (A)
14. Rare
(A) malicious (B) abstruse (C) relentless (D) overpowering
Ans: (B)
15. Felicity
(A) Anger (B) Millionaire (C) Bliss (D) Feathery
Ans: (C)
Directions 6 to 20: Pick out from the words, given below each sentence, the
word which would complete the sentence correctly and meaningfully
1. I will have to wait on this side of the road until the policeman................. the traffic.
A) will stop B) would stop C) could stop D) stops
Ans: D
17. He went away before I.............. him the whole story.
A) can tell B) could tell C) would have told D) can told
Ans: B
18. He could never have known the secret unless you...................him.
A) had told it to B) tell it to C) told it to D) will tell it to
Ans: A
19. It was one of .....................biggest mistakes that I have ever made.
A) a B) that C) the D) my
Ans: C
20. She was one of the nicest persons...............taught us.
A) whoever B) that ever C) ever D) whomever
Ans: A
Directions for Questions 2 to25 :Each of the following sentences has been divided
into four parts There is an error in one of the parts Point out the part which has
an error
21. A) We cannot have peace until /
B) all the nations dealt with each other /
C) in a spirit of equality and friendliness /
D) and until we develop a new spirit of special integration.
Ans: A
22. A) That Christ is the embodiment of God on earth,/
B) and that he is the Son of God, /
C) are the belief /
D) of all orthodox Christians even today.
Ans: C
23. A) The thief, when he was passing through the gallery, /
B) was seen by the servant /
C) who fired him /
D) with a pistol in his hand
Ans: C
24. A) If my uncle will come today, /
B) I shall leave for Calcutta with him./
C) and stay therefore more than three months /
D) and learn Bengali.
Ans: A
25. A) I laboured very hard /
B) lest I may fail
C) or lose my division /
D) at the very end of my educational career.
Ans: A
O A. 1971 war changed the political geography of the subcontinent
B. Despite the significance of the event . there has been no serious book about the conflict
C. Surrender at Dacca aims to fill this gap
D. It also profoundly altered the geo-strategic situation in South-East Asia

1] ACBD 2] CADB 3] BADC 4] ADBC

Ans: 4
O A. Thus begins the search for relief: painkillers, ice, yoga, herbs, even surgery
B. Most computer users develop disorders because they ignore warnings like tingling
fingers, a numb hand or a sore shoulder
C. They keep pointing and dragging until tendons chafe and scar tissue forms, along with
bad habits that are almost impossible to chage
D. But cures are elusive , because repetetive stree injuries present a bag of ills that often
defy easy disgnosis.

1] BDAC 2] BADC 3] BCAD 4] ABCD

Ans: 3
O A. If you are used to having your stimulation come in from outside, your mind never
develops its own habits of thinking and reflecting
B. Marx thought that religion was the opiate, because it soothed people's pain and suffering
and prevented them from rising in rebellion
C. If Karl Marx was alive today, he would say that television is the opiate of the people.
D. Television and similar entertainments are even more of an opiate because of their
addictive tendencies.

1] BACD 2] ADBC 3] BDCA 4] CBDA

Ans: 4
O A. Then two astronomers-the German, Johannes Kepler, and the Italian, Galileo Galilei-
started publicly to support the Copernican theory, despite the fact that the orbits it
predicted did not quite match the ones observed.
B. His idea was that the sun was stationary at the centre and that the earth and the planets
move in circular orbits around the sun.
C. A simple model was proposed in 1514 by a Polish priest, Nicholas Copernicus.
D. Nearly a century passed before this idea was taken seriously.

1] CDBA 2] CBDA 3] BCAD 4] CADB

Ans:2
O A. By the time he got to Linjeflug four years later, he had learned many lessons, in fact,
he began his second stint as top dog by calling the entire company together in a hanger and
asking for help, a far cry from his barking out commands just 48 months back.
B. At SAS, he arrived at a time crisis.
C. This book is chock-a-block full of intrusive stories and practical advice, describing
Carton's activities at Vingresor (where he assumed his first presidency at age 32), Linjeflug,
and SAS in particular.
D. He began at Vingresor as an order giver, not a listener - neither to his people nor to his
customers and made every mistake in the book.

1] BADC 2] BACD 3] CBAD 4] CDAB

Ans:4
TCS COMPANY -I
(PAPER) TCS Online Test

The test was divided in three sections as:
Verbal reasoning (20 min - 32 Qs )
2 Quantitative aptitude (40 min - 3 Qs)
3 Critical reasoning (30 min - 3 passages - 12 Qs)

No Sectional Cut off.
No Negative Marks.
No Psychometric Test was there for us.


Verbal reasoning: (20 min 32 Qs)

Antonyms and Synonyms: (20) (I dont remember which were for antonyms and which were
for synonym)\
1. Effrontery -- extreme rudeness and lack of ability to understand that your behavior is not
acceptable to other people.

2. ERRATIC(SYNONYM)
a. Unromantic
b. Free
c. Popular
d. Steady
e. Unknown(answer)
3.what is the synonyms of RAPT
Ans:- concealed

4. SYNONYMS OF tranquil
Ans:- serene

5. Awry - skewed, crooked, wrong

. Repellent x attractive(antonym)

7. Raucous - rough, wild, hoarse, guttering(synonym)

8. Benign - kind, benevolent, compassionate

9. Pristine x sullied

10. Florid= Ornate, Showy(SYN)

11. Servility= Surrender(SYN)

12. Chide praise(ANT)

13. Stilted natural(ANT)

I dont Know all the words, but for scoring good marks in this section Refer Barrons 750
word list. As Well as refer DiscussionsWorldcom mug up all the synonyms and antonyms
of approx last 50-70 papers.
Definitely you will Score good marks here, Attempt all the question as there is no negative
marking.
I Think my score is good in this section.

There was paragraph making which i cud not do all the questions of that as it was very
tough, so i guessed the answers and hoped atleast few hits the target the. RC had a
passage from barons, i had practiced all the barrons RC and it helped .
For better score refer Barrons for this section.

Quantitative aptitude: -
FOR THE QUANT SECTION I WILL SUGGEST U TO SOLVE THE R.S.AGRAWAL AND A LOT
BETTER TO SOLVE THE LAST 3-4 YEARS QUESTION PAPERS FROM
THEDiscussionsWorldcom. It Will really Help You.

Sorry I dont remember the questions. But I am able to correctly solve the 33 questions and
remaining r the targeted hits.
I m Sending U some sample questions with their solutions, questions on my paper r like
these,

1) Complete the series 7, 9 , 13 , __ , 27, 37
Ans: 19

2) What is the largest prime number that can be stored in an 8-bit memory?
Ans: 127

3) Number of faces, vertices and edges of a cube
Ans : ,8,12(Sometimes they change the sequence as on my paper it comes as number of
vertices ,faces and edges , so first make a very close look to the question and decide the
sequence of values that matching the one of the options).

4).In a triangle which one is not possible. Sides are (5 ,5 ,5.), (5 ,4 ,5 ), (4 ,4, 9 ), (3,4
,5,)?
Ans: (4,4,9). (HINT : for a triangle sum of smaller 2 sides should be greater than the other
one which is larger)

5) Match the following:
1. Male - Boy a. A type of
2. Square - Polygon b. A part of
3. Roof - Building c. Not a type of
4. Mushroom - Vegetables d. A superset of
Ans: 1- d, 2- a, 3- b, 4- c

) Given $ means Tripling and % means change of sign then find the value of $%$-%$%
?
Ans : -72

8) In a two-dimensional array, X (9, 7), with each element occupying 4 bytes of memory,
with the address of the first element X (1, 1) is 3000, find the address of X (8, 5).
Ans : 3212
(Hint : Every 1 deg longitude is equal to 4 minutes . If west to east add time else subtract
time)

9) Select the odd one out. a. ,v, b. Lisp c. Smalltalk d. Eiffel.
Ans: LISP

10) In which of the system, decimal number 194 is equal to 1234?
Ans: 5

11) A power unit is there by the bank of the river of 750 meters width. A cable is made from
power unit to power a plant opposite to that of the river and 1500mts away from the power
unit. The cost of the cable below water is Rs. 15/- per meter and cost of cable on the bank
is Rs.12/- per meter. Find the total of laying the cable.
Ans: 20250

12) Which of the following are orthogonal pairs? a. 3i+2j b. i+j c. 2i-3j d. -7i+j
Ans : (A) & (C).

13) If VXUPLVH is written as SURMISE, what is SHDVD?
Ans : PEASA

14) If A, B and C are the mechanisms used separately to reduce the wastage of fuel by
30%, 20% and 10%. What will be the fuel economy if they were used combined.
Ans : 20%

15) A can copy 50 papers in 10 hours while both A & B can copy 70 papers in 10 hours.
Then for how many hours required for B to copy 2 papers?
Ans : 13

1) Find the singularity matrix from a given set of matrices? (Hint determinant(A)=0)

17) units of basic quantities :
(energy time time)/(mass dist) = distance

18) For a round trip, a car used 4 1/2 gallons of gasoline. If it used 1/4 more gasoline going
than coming back, how much gasoline was used coming back?
Ans: 2 gallons
19)A shopkeeper bought a watch for Rs.400 and sold it for Rs.500.What is his profit
percentage?
Ans. 25%
20)g[0]=1,g[1]=-1,g[n]=2g[n-1]-3g[n-2] then calculate g[4]=
21) .The ratio of boys to girls is :4.0% of the boys and 40% of girls take lunch in the
canteen.What % of class takes lunch?
Ans) 52%
22) Which of the following set of numbers has the highest Standard deviation?
a)1,0,1,0,1,0 b) -1, -1, -1, -1, -1, -1
c)1,1,1,1,1,1 d) -1,1,-1, 1,-1, 1
Ans : D
23) (momentum velocity)/(force time) = velocity
24-27) there were 3 questions on venn diagrams (like 20 spek eng,30 speak hindi 3 speak
both and so on..)
28). The size of a program is N. And the memory occupied by the program is given by M =
square root of 100N. If the size of the program is increased by 1% then how much memory
now occupied?
Ans. 0.5%.
29).Odd one out:
a. Linux b.windows NT c. $" server d. Unix
30). In the word ORGANISATIONAL, if the first and second, third and forth, forth and fifth,
fifth and sixth words are interchanged up to the last letter, what would be the tenth letter
from right?
31). If TAFJHH is coded as RBEKGI then RBDJK can be coded as?
32). FIND THE MISSING NO. IN SERIES ?
9 10 11 13 15 ? 21 28
Ans: 19
33). In the word CHAMPIONSHIP, if the first and second, third and forth, fifth and sixth, etc
words are interchanged, what would be the 8th letter from right? (Ans: `I)
34). If A, B, C are the mechanisms used separately to reduce the wastage of fuel by 30%,
40%, 10%.What will be the fuel economy if they were used combine?
1.8.4 2.2.2 3.58 4.27
(Ans- 2.2) sol: (70/100)(0/100)(90/100)100=37.8 Eco = (100-37.8) =2.2
35). What is the value of the following expression M(373,5)+T(7.7)+R(4.4)-T(3.)
Where M- MODULAS R- ROUNDOFF T- TRUNCATE
(Ans : 11) Sol: 3 + 7+ 4 - 3 ==11

3). Match the following (this type of question but not same)
A B
1. Mammal, cow ---> a. A type of
2. Snake reptile ---> b. A part of
3. Roof - Building ---> c. Not a type of
4. Mushroom - Vegetables ---> d. A superset of
(Ans: 1-d, 2-c, 3-b, 4-a)
37). If G(0) = -1 G(1)= 1 and G(N)=G(N-1) - G(N-2) then
What is the value of G()?(Ans: -3)

Sol: g(2)=g(1)-g(0) ==1-(-1)=2, similarly g(3),.g();
ONCE DO R.S.AGGARWAL (QUANT) (arithmetic section)
GOOD FOR U.
38). Which set of data exhibits a higher Standard Deviation?
(a) 9, 0, -9, 9, 0, -9 (b) 9, 9, 9, 0, 9, 9 (c) -9, -9, -9, -9, -9, -9 (d) 9, 9,
9, 9, 9, 9 (e) 9, -9, 9, -9, 9, -9
Ans: e....take mean(sumof all/no.of elmnts),take difference of all frm mean,now apply
formula (s.d)^2=(sum of (diff)^2/n)
39). In Chennai , temperature at noon varies according to -t^2/2 + 8t + 3, where t is
elapsed time. Find how much temperature more or less in 4pm to 9pm.
Ans. (put 9 & 4in eq. n subtract )
40). Largest prime no 3 digit
Ans: 997
41). Find the value of @@+25 - ++@1, where @ denotes "square" and + denotes "square
root".
Ans. 21
42). (MomentumVelocity)/(Acceleration distance ) find units.
Ans. Mass
43). In a two-dimensional array, X(9,7), with each element occupying 2 bytes of memory,
with the address of the element X(1,1) is 3000,
find the address of X(8,5)?
Ans: 310
44). Find the fourth row, having the bit pattern as an integer in an 8-bit computer, and
express the answer in its decimal value.
A 0 0 0 0 1 1 1 1
B 0 0 1 1 0 0 1 1
C 0 1 0 1 0 1 0 1
(A OR(B AND C)) ?
Ans: B AND C
1 AND 1=1 else 0
1 OR 0/1=1
So B AND C-- 00010001 This OR A->00011111
45). An aircraft takes off from A (89o N Lat, 20o E ) at .00 AM local time to B (50o S , 40o
W ). If the flying time is 10 hours what is the local time of landing at B?
Ans:12:00pm
dont take care of N & S.E & W matters only.
1o change == 4mins
Here 20 o+40 o=0 o
So 04=240 mins..4hrs
If time change not considered then After 10 hrs it il reach at the same palce after
:00+10:00=4:00pm
Going frm east to west so substract 4hrs frm 4:00pm.
4:00pm-4hrs=12:00pm
(if plane is going frm west to east then then Add those hrs)
4). Select the odd one out
a. SMTP b.WAP c. SAP d.ARP
Ans : c
47). select odd one- sql,db2,sybase,http {ans-htttp}
48). select odd one-sybase,db2,oracle,unix {ans-unix}
49). A can copy 50 papers in 10 hrs while A &B together can copy 70 papers in 10 hrs.how
many hrs will be required for B to copy 2 papers?
Ans: 13hrs
50). Find eqn of line having intercepts (0,3) & (-2,0)
Ans Y=(3/2)X + 3
Formula x/a + y/b = 1
51). ) if n=81251 which one is not int?
a>n/13 b>n/244 c> n/72
Ans: c
52). Which shape will be obtained by using the following values for X and Y
X 0 10 100 1000 9999
Y 0.00001 1.02 1.72 3.00 4.72
Ans: Y= log10(X)
53). If log 0.317=0.3332 and log 0.318=0.334 then find log 0.319 ?
Sol) log 0.317=0.3332 and log 0.318=0.334, then
log 0.319=log0.318+(log(0.318-0.317)) = 0.339
54). A box of 150 packets consists of 1kg packets and 2kg packets. Total weight of box is
24kg. How many 2kg packets are there ?
Sol) x= 2 kg Packs
y= 1 kg packs
x + y = 150 .......... Eqn 1
2x + y = 24 .......... Eqn 2
Solve the Simultaneous equation; x = 114
so, y = 3
Ans : Number of 2 kg Packs = 114.
55). A plane moves from 9N40E to 9N40W. If the plane starts at 10 am and takes 8
hours to reach the destination, find the local arrival time ?
Sol) Since it is moving from east to west longitide we need to add both
ie,40+40=80
multiply the ans by 4
=>804=320min
convert this min to hours ie, 5hrs 20min
It takes 8hrs totally . So 8-5hr 20 min=2hr 40min
So the ans is 10am+2hr 30 min
=>ans is 12:30 it will reach
5). A fisherman's day is rated as good if he catches 9 fishes, fair if 7 fishes and bad if 5
fishes. He catches 53 fishes in a week n had all good, fair n bad days in the week. So how
many good, fair n bad days did the fisher man had in the week
Ans:4 good, 1 fair n 2 bad days
Sol) Go to river catch fish
49=3
71=7
25=10
3+7+10=53...
take what is given 53
good days means --- 9 fishes so 53/9=4(remainder=17) if u assume 5 then there is no
chance for bad days.
fair days means ----- 7 fishes so remaining 17 --- 17/7=1(remainder=10) if u assume 2
then there is no chance for bad days.
bad days means -------5 fishes so remaining 10---10/5=2days.
Ans: 4 good, 1 fair, 2bad. ==== total 7 days.
x+y+z=7--------- eq1
9x+7y+5z=53 -------eq2
multiply eq 1 by 9,
9x+9y+9z=35 -------------eq3
from eq2 and eq3
2y+4z=10-----eq4
since all x,y and z are integer i sud put a integer value of y such that z sud be integer in eq
4 .....and ther will be two value y=1 or 3 then z = 2 or 1 from eq 4
for first y=1,z=2 then from eq1 x= 4
so 94+17+25=53.... satisfied
now for second y=3 z=1 then from eq1 x=3
so 93+37+15=53 ......satisfied
so finally there are two solution of this question
(x,y,z)=(4,1,2) and (3,3,1)...

57) Y catches 5 times more fishes than X. If total number of fishes caught by X and Y is 42,
then number of fishes caught by X?
Sol) Let no. of fish x catches=p
no. caught by y =r
r=5p.
r+p=42
then p=7,r=35

58) Three companies are working independently and receiving the savings 20%, 30%, 40%.
If the companies work combinely, what will be their net savings?
suppose total income is 100
so amount x is getting is 80
y is 70
z =0
total=210
but total money is 300
300-210=90
so they are getting 90 rs less
90 is 30% of 300 so they r getting 30% discount

59) The ratio of incomes of C and D is 3:4.the ratio of their expenditures is 4:5. Find the
ratio of their savings if the savings of C is one fourths of his income?
Sol) incomes:3:4
expenditures:4:5
3x-4y=1/4(3x)
12x-1y=3x
9x=1y
y=9x/1
(3x-4(9x/1))/((4x-5(9x/1)))
Ans:12/19
0).The size of a program is N. And the memory occupied by the program is given by M =
square root of 100N. If the size of the program is increased by 1% then how much memory
now occupied ?
Sol) M=sqrt(100N)
N is increased by 1%
therefore new value of N=N + (N/100)
=101N/100
M=sqrt(100 (101N/100) )
Hence, we get M=sqrt(101 N).
1). A person had to multiply two numbers. Instead of multiplying by 35, he multiplied by
53 and the product went up by 540. What was the raised product?
a) 780 b) 1040 c) 1590 d) 1720
Sol) x53-x35=540=> x=30 therefore, 5330=1590 Ans

Just Go through the previous year question papers and solve all the questions. The pattern
of the questions r same only the values are changed, so go through and solve those
questions.
Wishing U all the best.
Critical reasoning (30 min - 3 passages - 2 Qs)
For the Critical reasoning section refer barons 12th edition critical reasoning and solve all
the model question papers that are given there. As try to mug up the answers.

Out of the 12 questions 8 are from the model test papers and other 4s are easily solvable.
As I think all the 12 answers of this section is of mine are correct.

RESULT OF THE ONLINE TEST DECLARED AT EVENING 09:50 Approx.

I am one of the candidates who was short listed for the Technical Interview.

As it is was very late. So they Call Us at the next day at 1:00 PM sharp.

Technical Interview
TechnAs the technical round was supposed to take place at 1:00 PM, but unfortunately the
technical interviews for the our college starts at 07:00 pm, My Technical Interview takes
place at 08:30 Pm.
I have to wait for the long time at the door of the Interviewer cabin as the Interviewer is
writing something on the paper.
As I entered the room.
ME:-Good Evening Sir.
SIR: Have seat.
ME: Thank you very much sir, and takes the seat.
SIR: Tell me about yourself.
Me: I said my name, my college name, my degree, about my family and my hobbies, some
of my positive qualities, my role model.
SIR: Can U write the program of string palindrome in C++.
Me: Yes Sir I Can. He gave me a paper , And Then I wrote The whole Code for the String
palindrome using pointers. Please Be specific and dont waste time , if u dont know the
code, just say sorry sir/mam.
SIR: Can U write the program
1 4.
2 3
3 2
4 1.
Me: Yes Sir I Can. And then I wrote the code , It just take two for loops(nested) one move
from 1 to 4 and other from the 4 to 1.
Sir: What are your core (favorite) subjects in MCA?
Me: I said Computer Networks sir.
Sir:-Oh Really Computer Networks.
Me:-Yes Sir, I think as I said computer networks that was out of the leak no MCA students
before me told that answer(As I think), But I gave that answer because I m having very
good knowledge about the Computer networks both theoretically as well as practically.
(Please dont bluff).
SIR:- OK, Brashaket Tell me What Is WINC?
ME:- It was an blunder for me as said Computer networks , and I dont know the answer of
that question . But I m still confident and simply say very politely sorry sir.
SIR:-OK, What is URL?
Me- URL Means Uniform Resource Locator. And then told all the methods and the whole
working of URL.
SIR:- What is DNS.
ME:-DNS means (Domain name systems) and then told whole working of the DNS.
SIR:-Can U tell me what is SMTP?
ME:-It means Simple Mail Transfer Protocol. And told the working of the SMTP.
SIR:-What is Cryptography?
ME:-Told.
SIR:- Sir: What are the different types of Topologies?, what are merits &demerits of each?
Me:-BUS,STAR,MESH,RING,TREE,PARTIAL MESH topology. And told the merits and demerits
of all.
Sir: what is the Topology used in Ur college?
ME:- I said it is star topology.
SIR:-What r the different networking devices?
ME:- Sir, HUB, Switch, #outers, Bridges, etc.
SIR:-What is the difference between the switch, routers and bridges?
Me:-I Told.
SIR:- Then He made a very close look to my resume. (Be prepare with each and every
aspect of your resume and dont write any single thing that u dont know.). Then He asked
me About IBM DB2, as I have wrote IBM DB2 in the technical skills section in my resume.
ME:-Told.(As I did the training of DB2 in our College).
SIR:-Ok Brashaket. Then He asked me about my projects? Please explain?
Me:-As I wrote the 2 of my projects their. I Explained. At that time when I was explaining I
asked him for a blank paper , and I explained that on a paper. My Project is on Data
Dictionary of Oracle, SQL-Server and MS-ACCESS. It was titled Data dictionary
management system. It eliminate the writing of complex queries.
I told Each and every aspect of the project from the technologies to the reports.(the
explanation took approx 15-20 minutes.
SIR:- He said Very good Brashaket. And then he asks me that R U really interested in
development ?
Me: I told yes very much sir.(I dont know why he asks that question).
SIR:-Ok Brashaket You may leave now . Then He Offer shake hands to me ?
Me:-Shake hand with him and Said thank you sir? Have a good day. And leave the room.
My technical round took about 40-45 minutes .
At 09:30 PM the results of the technical round was declared , My name is in the list of the
selected candidates . and then they gave us a form to fill. And as it was very late they told
us to come tomorrow at 09:00 AM sharp.
As 138 students of my college appeared for the tech. round . out of 138 , 70 are short listed
for the Managerial reasoning (M.R.) round.
Speak very polite and be confident in the interview Don't try answering the
question that you really don't know Be very careful with the resumes keep it as
simple as you can .

MR Interview
At 09:AM sharp the M.R. interviews of the students of my college had started.
My Number came at approx 11:00 AM.
Me:- I have to go to the panel number 9.
Me:- Knocked the door. And said Sir, May I come in ? There is an gentle man of age approx
50.
Sir:-Please come in?
Sir:- Have Seat?
Me:- I took and told thank u very much sir.
Sir:- He asked me what is the correct pronunciation of your name?
Me:- I told.
Sir:- Then he asked me that what is the meaning of ur name?
Me:-Told (It is one the name of the lord Shiva).
Sir:- Ok Brashaket . Tell me about yourself?
Me:- I told blah blah ?
Sir:- Tell me something about the family background?
Me:- Told.
Sir:- Then He gave me a puzzle to solve.
Me:- I am able to solve that one perfectly.
Sir:- What do u know about the new technology?
Me:-I told abt the nano technology.
Sir:-Then He asked me about the criteria behind launching the new OS?
ME:- I Told. That For the better appearance and for the better GUI and For the
Better$ecurity the new OS is lunched.
Sir:-he seem to looked that he is not satisfied with the answer and then he told me that on
the basis of these thing u will launch a new OS.
Me:-yes Sir, Try to explain more, but he is not satisfied, as I think he is checking my patient
. Again he was tried to frustrate me but I said sir in my opinion the new OS are launched on
these basis. And the other third party s/w can easily be installed.
Sir:- Ok , Any Questions Brashaket?
Me:-Sir what kind of preparation I have to do before joining the TCS training program?
Sir:- Just learn new and more languages and technologies.
Sir:- Thank U very much Brashaket?
Me:- Thank U very much sir , It was really nice Meeting U?
Sir:- Ok, You may leave now?
Me:- Leave and told again thank u sir and have a good day sir. and left.
My M.R. Interview lasting approx for about 35 minutes.
I m very scared as the persons whose name is called think that he has cleared the MR round
but some of the candidates r called the interviewers who took the MR interviews shake
hands with them and then said "You may leave for home now. It was a blunder for the
person that think that he is selected but the result is that he is rejected.
After 2 hour I was Called for the HR round. Luckily I m selected for the HR Round.
HR Interview
Again I have to wait for a long time as the interviewer is writing something. But he was as
decent personality. He said sorry for that, that u have to wait for a long time, I told Its OK,
No problem sir.
SIR:- May I know Ur good name please?
ME:-Told.
SIR:-What is the meaning of the name?
Me:-Told
Sir:- How was the Day Brashaket ?
Me:- It was Nice Sir.
Sir:- How was Previous two interviews?
Me: Both of the interviews are Very nice sir.
SIR:-Tell me something abt ur family background?
Me:-Told
SIR:- How many siblings do u have?
Me:- Told
Sir:- what Ur father is doing?
Me:- Told( As my father is working in a cement industry In Maihar)
Sir:- He asked me abt the other cement plants there?
Me:-Told.
Sir:- He told that satna is known for the cement industries naa , can u tell me the names of
the cement industries situated their?
Me:-Told.
Sir:- Ok Brashaket Suppose U have nine dots and u have to connect these dots with the
help of the four line and u dont have to lift ur pen. Can U?
Me:- I said I will Try.
Sir:- OK , Suppose that U have a pencil and U have to cross a river with the help of that
pencil. How you will cross?
Me:- I gave the answer like this
Sir there might be the probability that there is no water is available on the river. Or we can
map the depth of the river with the help of that pencil by dipping it in the river and if the
river is not so deep then we can cross the river easily.
Sir:-Oh, really?
Me:-Yes sir , As I think.
Sir:- Then He gave me a situation , that suppose u r appointed in a team in a s/w company.
The team consist of the 4 members. He said U belongs to a very low ranked institute like
TIT suppose one of the member of the team is from IIT kanpur, one from IIT Bombay and
one from the Other IITs and they think that u belongs to a very low standard institute so
they r not coordination with u , in that situation what u will do?
Me:- Told , I am trying to satisfy him and I am able to satisfy him with my answer.
Sir:- Is their any pan for the higher studies?
Me:- No Sir, As I m going to complete My PG program and Now I wanted do job now.
Sir:- Tell me something about ur weaknesses?
Me:-Told.(BE Prepare and dont bluff).
Sir:-Asked some more questions abt the my strengths and weaknesses?
Me:-I told.
Sir:- Is there any location constraints?
Me:- No Sir, Yoy can through me in any part of the the world.
Sir:-laughed. Are U ready to sign the bond ?
Me:-Yes sir.
Sir:- All The Best Brashaket.
Me:-Thank U Sir.
Sir:-Ok Brashaket , Thanks and U may leave Now.
Me:-Thank U very much Sir and have a good day sir and left.

My H.R. Interview lasting approx for about 45 minutes.
After Waiting for a long time the final result of the selection process is declared at 10:00 PM
. And Bhagwan ki Kripa aur maa & papa ke ashirwaad aur meri mehnat ke karan I Was One
of the candidates who is declared as a part of TCS.
Finally 5 Students r selected from our college, but only 4 r from the MCA. Remaining 52 r
from B.E..
It was the happiest moment in my life.
Friends any one can easiliy grab TCS if you are technically strong in 3 of your department
paper and with 100% confidence and positive attitude. GOOD LUCK.


TCS -II
)APTITUDE TEST:
Questions = 82 ; time limit = 90 minutes. no negative marking. IIline (p,per & pen) test
,nd , P$CH%# %$% ,lso.
$ection 1: VERB ( 32 Questions ,20 minutes )
Directions for questions -0:Find the synonyms of the following words
1. ADAGE
A. advice B. proverb C. enlargement D. advantage
Ans: B

2. TO DISPEL
A. to dissipate B. to dissent C. to distort D. to disfigure
Ans: A

3. ERRATIC
A. unromantic B. unknown C. popular D. steady
Ans: B

4. TO MERIT
A. to embrace B. to devote C. to deserve D. to combine
Ans: C

5. RAPT
A. lively B. concealed C. engrossed D. prototype
Ans: B

. TO HEAP
A. to pile B. to forbid C. to proceed D. to share
Ans: A

7. OVULATE
A. penury B. immunize C. fertilize D. reproduce
Ans: C
8. Motley
A. Uniform B. homogenous C. monochrome D. Assorted
Ans: D
9. mitigate
A. palliate B. Aggravate C. exacerbate D. None of these
Ans: A
10. Compunction
A. #egret 8. scruple C. qualm 0. None of these
Ans: D
Directions for questions -20:Find the Antonyms of the following words
11. Moribund
A. declining B. dilapidated C. waning D. thriving
Ans: D
12. Repudiate
A. reject B. disclaim C. acknowledge D. renounce,
Ans: C
13. Inundate
A. starve B. swamp C. deluge D. drown
Ans: A
14. Tenacity
A. obstinacy B. irresolution C. firmness D. resolve
Ans: B
15. Sobriety
A. flIppancy 8. temperance C. moderatIon 0. soberness
Ans: A
1. Hidebound
A. narrow-minded B. conservative C. prejudiced D. broad-minded

Ans: D
17. Nebulous
A. vague B. imprecise C. precise D. hazy
Ans: C
18. Debacle
A. catastrophe B. fiasco C. success D shambles,
Ans: C
19. Candid
A. frank B. guarded C open D. truthful
Ans: B
20. Dangle
A. stick up B. sway C. suspend D. droop
Ans: A
Directions for Questions 2-26: Read the passage and answer the questions that
follow on the basis of the information provided in the passage
Educational planning should aim at meeting the educational needs of the entire population
of all age groups. While the traditional structure of education as a three layer hierarchy
from the primary stage to the university represents the core, we should not overlook the
periphery which is equally important. Under modern conditions, workers need to rewind, or
renew their enthusiasm, or strike out in a new direction, or improve their skills as much as
any university professor. The retired and the aged have their needs as well. Educational
planning, in their words, should take care of the needs of everyone.
Our structures of education have been built up on the assumption that there is a terminal
point to education. This basic defect has become all the more harmful today, A UNESCO
report entitled 'Learning to Be' prepared by Edger Faure and others in 1973 asserts that the
education of children must prepare the future adult for various forms of self-learning. A
viable education system of the future should consist of modules with different kinds of
functions serving a diversity of constituents. And performance, not the period of study,
should be the basis for credentials. The writing is already on the wall.
In view of the fact that the significance of a commitment of lifelong learning and lifetime
education is being discussed only in recent years even in educationally advanced countries,
The possibility of the idea becoming an integral part of educational thinking seems to be
afar cry. For, to move in that direction means such more than some
simple rearrangement of the present organisation of education. But a good begining can be
made by developing open university programmes for older learners of different categories
and introducing extension services in the conventional colleges and schools. Also these
institutions should learn to co-operate with the numerous community organizations such as
libraries, museums, municipal recreational programmes, health services etc.
21. What is the main thrust of the author ?
A) Traditional systems should be strengthened
B) Formal education is more important than non-formal
C) One should never cease to learn
D) It is impossible to meet the needs of everyone
E) There is no substitute for the extent system of education
Ans: C
22. Which of the following best describes the purpose of the author ?
A) To criticise the present educational system.
B) To strengthen the present educational practices
C) To support non-conventional educational organisations.
D) To present a pragmatic point of view
E) None of these
Ans : D
23.Which of the following is most nearly the same in meaning as the word 'meeting' as used
in the passage.
A) Approaching B) Contacting C) introducing D) representing E) satisfying
Ans: E
24. Which of the following is most opposite in meaning to the word 'integral' as used in the
passage.
A) essential B) independent C) major D) minor E) unwilling
Ans: B
25. According to the author, what should be the basis for awarding credentials?
A) Duration of the course
B) Competence of the course teachers
C) Diversity of the topics covered
D) Real grasp of matter or skill
E) Participation in community activities
Ans: D
2. According to the author, educational plan should attempt to
A) train the people at the core
B) encourage conventional schools and colleges
C) decide a terminal point to education
D) overlook the people on the periphery
E) fulfil the educational needs of everyone.
Ans : A
Directions 2-32 : Pick out the most effective word from the given words to fill in
the blank to make the sentence meaningfully complete
27. .............you meet my son in the market, ask him to come home at once
A) Will B) While C) Should D) Would
Ans : C
28. Sanjay was sure..............getting a first class in the examination
A) at B) of C) on D) about
Ans : B
29. She seems offended .................my remarks
A) with B) for C) upon D) at
Ans : D
30. The mounting pressure was so overwhelming that he ultimately.................to her wish
A) agreed in B) cowed in C) gave in D) yielded in
Ans : C
31. He has no objection.................my proposal
A) to B) for C) in D) towards
Ans: A
32. Wash your hands.............water
A) from B) with C) by D) in
Ans: B
$ection 2: QU%%%VE/OGC RE$OG ( 38 questions , 40 minutes )

1. Two Pencils 8 cents 5 Pencils cost
Ans: 20 cents
2. II 2x-y4 then 6x-3y?
(,)15 (b)12 (c)18 (d)10

ns. (b)
3. Pipe A c,n Iill in 20 minutes ,nd Pipe B in 30 mins ,nd Pipe C c,n empty the s,me in 40
mins. II ,ll oI them work
together, Iind the time t,ken to Iill the t,nk
(,) 17 1/7 mins (b) 20 mins (c) 8 mins (d) none oI these
ns. (a)
4. %hirty men t,ke 20 d,ys to complete , job working 9 hours , d,y. How m,ny hour , d,y
should 40 men work to
complete the job?
(,) 8 hrs (b) 7 1/2 hrs (c) 7 hrs (d) 9 hrs
ns. (b)
5. A go,t is tied to one corner oI , squ,re plot oI side 12m by , rope 7m long. Find the ,re, it
c,n gr,ze?
(,) 38.5 sq.m (b) 155 sq.m (c) 144 sq.m (d) 19.25 sq.m
ns. (a)
. r. $h,h decided to w,lk down the esc,l,tor oI , tube st,tion. He Iound th,t iI he w,lks
down 26 steps, he
requires 30 seconds to re,ch the bottom. However, iI he steps down 34 st,irs he would only
require 18 seconds
to get to the bottom. II the time is me,sured Irom the moment the top step begins to descend to
the time he steps
oII the l,st step ,t the bottom, Iind out the height oI the st,ir w,y in steps?
ns.46 steps.

7. %he ,ver,ge ,ge oI 10 members oI , committee is the s,me ,s it w,s 4 ye,rs ,go, bec,use ,n
old member h,s
been repl,ced by , young member. Find how much younger is the new member?
ns.40 years.
8. ABC is ,n isosceles tr,pezoid ,nd ACD is , rect,ngle. AB 10 ,nd C 20. Wh,t is the
length oI A?

ns. E 10.
9.%hree c,rds ,re dr,wn ,t r,ndom Irom ,n ordin,ry p,ck oI c,rds. Find the prob,bility th,t they
will consist oI ,
king, , queen ,nd ,n ,ce.
ns. 64/2210.
10. A number oI c,ts got together ,nd decided to kill between them 999919 mice. very c,t
killed ,n equ,l number
oI mice. ,ch c,t killed more mice th,n there were c,ts. How ,ny c,ts do you think there
were ?
ns. 991
11. If Log2 x - 5 Log x + = 0, then what would the value / values of x be?
Ans x = e2 or e3
12. The square of a two digit number is divided by half the number. After 3 is added to the
quotient, this sum is
then divided by 2. The digits of the resulting number are the same as those in the
original number, but they are
in reverse order. The ten's place of the original number is equal to twice the difference
between its digits.
What is the number?
Ans 46
13. A monkey starts climbing up a tree 20ft. tall. Each hour, it hops 3ft. and slips back 2ft.
How much time would it
take the monkey to reach the top?
Ans8 hours
14. What is the missing number in this series?
8 2 14 11 ? 14 18 12
Ans 9
15. A certain type of mixture is prepared by mixing brand A at Rs.9 a kg. with brand B at
Rs.4 a kg. If the mixture
is worth Rs.7 a kg , how many kgs. of brand A are needed to make 40kgs of the
mixture?
Ans Brand A needed is 24kgs
1. A wizard named Nepo says "I am only three times my son's age. My father is 40 years
more than twice my age.
Together the three of usare a mere 1240 years old." How old is Nepo?
Ans 360 years old
17. One dog tells the other that there are two dogs in front of me. The other one also
shouts that he too had two
behind him. How many are they?
Ans Three
18. A man ate 100 bananas in five days, each day eating more than the previous day.
How many bananas did he
eat on the first day?
Ans Eight
19. If it takes five minutes to boil one egg, how long will it take to boil four eggs?
Ans Five minutes
20.Daal is now being sold at Rs. 20 a kg. During last month its rate was Rs. 1 per kg. By
how much percent
should a family reduce its consumption so as to keep the expenditure fixed?
Ans 20 %
21. Can you find out what day of the week was January 12, 1979?
Ans Friday
22. From 5 different green balls, four different blue balls and three different red balls, how
many combinations of
balls can be chosen taking at least one green and one blue ball?
Ans 320
23. Three pipes, A, B, & C are attached to a tank. A & B can fill it in 20 & 30 minutes
respectively while C can
empty it in 15 minutes. If A, B & C are kept open successively for 1 minute each, how
soon will the tank be
filled?
Ans 6 minutes
24. A person walking 5/ of his usual rate is 40 minutes late. What is his usual time?
Ans 3 hours 20 minutes
25. A garrison of 3300 men has provisions for 32 days, when given at a rate of 850 grams
per head. At the end of
7 days reinforcement arrives and it was found that now the provisions will last 8 days
less, when given at the rate
of 825 grams per head.How, many more men can it feed?
Ans 00 men
2. In the given figure, PA and PB are tangents to the circle at A and B respectively and the
chord BC is parallel to
tangent PA. If AC = cm, and length of the tangent AP is 9 cm, then what is the length
of the chord BC?
Ans BC = 4 cm
27. A boat travels 20 kms upstream in hrs and 18 kms downstream in 4 hrs. Find
the speed of the boat in still
water and the speed of thewater current?
(a) 1/2 kmph (b) 7/12 kmph (c) 5 kmph (d) none of these
Ans (b)

28. Find the smallest number in a GP whose sum is 38 and product 1728
(a) 12 (b) 20 (c) 8 (d) none of these
Ans (c)
29. If x=y=2z and xyz=25 then what is the value of x?

(a)12 (b)8 (c)1 (d)

Ans (b)

30. (1/10)18 - (1/10)20 = ?
(a) 99/1020 (b) 99/10 (c) 0.9 (d) none of these
Ans (a)
31.A train running at 72 kms per hour crosses a coconut tree standing by the side of the
track in 7 seconds. The
length of the train is:
(A) 104 metres (B) 140 metres (C) 504 metres (D) 540 metres
Ans: B
32. If 12 men and 1 boys can do a piece of work in 5 days and 13 men and 24 boys can do
it in 4 days, the ratio
of daily work done by a man to that done by a boy is:
(A) 1 : 3 (B) 1 : 2 (C) 2 : 1 (D) 3 : 1
Ans: C
33. A cistern is filled by a tap in 3 1/2hours. Due to a leak in its bottom, it takes 12 hour
longer to fill. If cistern is
full, how long will it take to leak to empty it?
(A) 7 hours (B) 8 hours (C) 14 hours (D) 28 hours
Ans: D
34. A can do a piece of work in 7 days of 9 hours each whereas B can do the same work in
days of 7 hours each
How long will it take to complete the work together working 8 2/5 hours a day?
(A) 2 days (B) 3 days (C) 3 1/7 days (D) 4 2/5 days
Ans: B
35. A bus left Delhi for Ambala at 50 km/hr and turned over the same route at 40 km/hr.
Thus it took 1 hour more
on the return trip. The distance between Delhi and Ambala is:
(A) 200 kms (B) 180 kms (C) 400 kms (D) None of these
Ans: A
3. A man purchased a bag of rice containing 70 kgs for Rs 175. He sold it at the rate of Rs
2.75 per kg.
Find the profit and loss per cent.
(A) 10% profit (B) 10% loss (C) 12.5% profit (D) 12.5% loss
Ans: A
37. A shopkeeper gives 3 consecutive discounts of 10%, 15% and 15% after which he sells
his goods at a
percentage profit of 30.05% on the cost price. Find the value of the percentage profit
that the shopkeeper
would have earned if he had given discounts of 10% and 15% only:
(A) 53% (B) 2.5% (C) 72.5% (D) 8.%
Ans: A
38. A journey of 192 kms takes 2 hours less by a fast train than by a slow train. If the
average speed of the slow
train is 1 kmph less than that of a fast train, what is the average speed of the fast
train?
(A) 30 kmph (B) 48 kmph (C) 20 kmph (D) 25 kmph
Ans: B
$ection 3: CR%C RE$OG ( 2 questions , 30 minutes)
Directions -2: Answer the questions given below the passage or statement as
true, false or can't say
PASSAE : In the past helicopters were forced to ground or crash because of the formation
of the ice on the rotors and engines. A new electronic device has been developed which can
detect the water content in the atmosphere and warns the pilot if the temperature is below
freezing temperature about the formation of the ice on the rotors and wings.
Answer questions 17-20 based on passage
1.The electronic device can avoid formation of the ice on the wings
AnsFalse
2. There will be the malfunction of rotor & engine because of formation of ice
AnsTrue
3. The helicopters were to be crashed or grounded
AnsTrue
4. There is only one device that warn about the formation of ice
AnsTrue
PASSAE : Human existence is suspicious of arbitrary divide between concise and
unconcise. The concise world invades shape activity of the unconcise, while many of great
activity of humanity waking as whole or partially improved by dreams. Even it could be
ignored that dreams precede exceptional such a dichotomy could not be drawn as the
influence of dream on waking state would remain unclear. But as yet no company rebuilt
exists to record the substitute of prenatal dreaming.
Answer questions 5- 8 based on passage

5. Sleepy can be creative state
Ans True
. It is difficult to tell whether a sleeper is dream or not
Ans True
7. If we know what babies would dream about before they are born we could show that the
concise and unconcise
mind influence on one another .
Ans Can't say
8. It is untrue claim that concise and unconcise world never impinge one another
AnsTrue
PASSAE : Although invaders represent a threat to the conservation of flora and fauna,
there are two special cases in which invasion have been deliberately brought about. One is
the desire to control presents by natural predators, which may have to be brought in from
other countries. The second is releasing organisms into the wild (or on to farms, from
which they might escape) that are completely novel, because they have been genetically
engineered. There is nothing intrinsically sinister about engineered organisms, but any
novelty must be regarded as a potential invader.
Answer questions 9-12 based on passage
9.Pest control does not threat the conservation of flora and fauna.
AnsTrue
10.Genetically engineered organisms must always be regarded as potentially dangerous.
Ans False
11.Natural predators are work harmful than pests.
Ans True
12.Genetically engineered organisms escaped from the farm, they will be pose a threat to
wildlife.
Ans True

TCS -III
)APTITUDE TEST:
Questions = 82 ; time limit = 90 minutes. no negative marking. IIline (p,per & pen) test
,nd , P$CH%# %$% ,lso.
$ection 1: VERB ( 32 Questions ,20 minutes )
Directions for questions -0 :Find the synonyms of the following words
1. ACUMEN
A. exactness B. potential C. shrewdness D. bluntness
Ans: C
2. DISCRETION
A. prudence B. consistency C. precipice D. disturbance
Ans: A

3. ORDAIN
A. arrange B. command C. contribute D. establish

Ans: B

4. FLORID
A. ornate B. thriving C. artistic D. elegant
Ans: A
5. PENITENCE
A. liking B. insightful C. compunction D. penetrable
Ans: C

. WHET
A. stimulate b. humorous c. inculate d. dampen

Ans: A
7. LATITUDE
A. scope B. lamentation C. globule D. legislature
Ans: A
8. dispel
A. accumulate B. collect C. garner D. deploy
Ans: D
9. Orthodox
A. heterodox B. unconventional C. conventional D. untraditional
Ans: C
10. Alienate
A. be friendly B. estrange C. disarm D. None of these
Ans: B
Directions for questions -20:Find the Antonyms of the following words
11 .Dogmatic
A. Arbitrary B. doctrinal C. unbending D. Flexible
Ans: D
12. Fallible
A. Perfect B Imperfect C. unsound D. mortal
Ans: A
13. Harbinger
A. Forerunner 8. herald C. potent 0. None of these
Ans: D
14. Intermittent
A. AlternatIng 8. sporadIc C. Constant 0. None of these
Ans: C
15. Pensive
A. ignorant B. Brooding C. pondering D. meditative
Ans: A
1. Concur
A. harmonIze 8. coIncIde C. assent D. Conflict
Ans: D
17.Furtive
A. Open B. Surreptitious C. clandestine D. None of these
Ans: B
18. Efface
A. Obliterate B. Engrave C. eradicate D. None of these
Ans: B
19. Pretentious
A. Ostentatious B. pompous C. Down-to-earth D. conceited
Ans: C
20. Veer
A. avert B. skew C. whirl Dstay
Ans: D
Directions for Questions 2-26: Read the passage and answer the questions that
follow on the basis of the information provided in the passage
Amnesty International's charge that 'tens of thousands' of political prisoners, including
prisoners of conscience, are languishing in Indian jails and that prisoners are routinely
tortured in this country has to be seen in a much wider context than the organisation's
annual report cares to do. In its overall appraisal of 151 countries, Amnesty has accused
112 of torturing prisoners, 3 of harboring prisoners of conscience, 1 of resorting to
political killings and 53 of detaining people without a trial. Of these
apparently overlapping categories, India seems to have been excluded from the list of the
1 which undertake political killings. The report has however, pointed out that scores of
people in India die of torture in police and military custody and that many also simply
disappear. Clearly, only a thin line separates the 1 charged with political murder from the
rest. Before coming to such conclusions, however, it may also be necessary to classify the
various countries according to their political systems. Torture by the security forces and
killings at the behest of the government make no difference to the victims whether they are
in a democratic country or a totalitarian one. It is also nobody's case that a democratic
country is lessculpable than a dictatorship in the event of human rights violations. But the
point perhaps still needs to be made that torture or 'disappearances' represent a failure of
the system in a democracy in contrast to being an integral part of state policy in a country
ruled by an autocrat who is answerable to no one.
India may be guilty of keeping 'tens of thousands' behind bars and of the other human
rights abuses mentioned by qualitatively different place from a totalitarian country. It is in
this respect that Amnesty has been lass than fair. It has chosen to ignore the distinctions
between the good, the bad and the ugly. The openness of Indian society will be evident to
anyone who spends half an hour in one of its chaotic market-places or visits the law courts
or watches a political rally or reads a newspaper or strikes up a conversation with any
person on the roads. There is no sense of fear in India, as in a dictatorship. There is also
scope for securing relief from the heavy-handed behaviour of the authorities, even if the
human rights commission has not yet lived up to expectations. Unless such points are
recognised, Amnesty's assessment will seem to be a dry recital of statistics which
maypillory India simply because of its larger population. Mercifully, Amnesty nowadays at
least notes that the terrorists also indulge in human rights violations and that India has to
cope with several insurgencies fomented by a country where the military does not always
seem to be under the control of the elected government. True, there is much that is the
way the terrorist challenge is activating the self-correcting mechanism within a democracy
and not merely on painting a grim, even biased picture.
21. In the report, India has been excluded from which of the following categories of
violating human rights?
A) Torturing prisoners
B) Detaining without trial
C) Political killings
D) harbouring prisoners of conscience
E) None of these
Ans: C
22. Which of the following is not true in the context of the passage ?
A) India is guilty of some human rights abuses
B) Amnesty International appraised all the democratic countries
C) There is overlapping of cases in the categories of human right abuses
D) India was one of the countries appraised by Amnesty International
E) The report notes that the terrorists also violate human rights
Ans: B
23. According to the passage, through which media of forum Amnesty International has
hurled the charges?
A) Seminar on Human rights
B) Its Regional Meet
C) Its annual Report
D) Its International Meet
E) None of these
Ans: C
24. The author of the passage
A) agrees with the report
B) disagrees with the report
C) disagrees that conditions of prisons in India is bad
D) supports the totalitarian approach
E) disagrees with report on terrorists
Ans: B
25. The Amnesty Internationals report is based on the information of how many countries ?
A) 3 B) 112 C) 131 D) 115 E) None of these
Ans: E
2. The author suggests classification of various countries on the additional dimension.
Which of the following is that dimension ?
A) Economic progress B) Human rights C) Industrial Progress
D) Political systems E) None of these
Ans: B
Directions 2-32 : Pick out the most effective word from the given words to fill in
the blank to make the sentence meaningfully complete
27. The boy you met yesterday is in class..................
A) ninth B) the ninth C) nine D) the nine
Ans : C
28. There was some confusion.................the agreement
A) on B) in C) around D) over
Ans : D
29. Speak loudly as he is slow.........................hearing
A) in B) about C) at D) of
Ans : C
30. Will you..................may dog while I am on tour ?
A) look out B) look up C) look on D) look after
Ans : D
31. Savitha is disgusted...................the habits of her husband
A) of B) from C) with D) at
Ans : C
32. Every man craves....................recognition
A) for B) about C) at D) after
Ans : D
$ection 2: QU%%%VE/OGC RE$OG ( 38 questions , 40 minutes )

1. If a + b + c = 0, then (a3 + b3 + c3) abc is equal to:
(A) 1 (B) 2 (C) 3 (D) 9
Ans : C
2.The difference between compound interest and simple interest for 3 years at 5% per
annum can be found out by
multiplying the principal by:
(A) 1.725 (B) 0.725 (C) 0.0725 (D) 0.00725
Ans : A
3.The simple interest on a sum of money is 1/9
th
of the sum and the number of years and
the rate per cent per
annum are equal. The rate per cent per annum is:
(A) 3 1/3 (B) 5 (C) 2/3 (D) 10
Ans : A
4.A man invested Rs 5,000 at some rate of simple interest and Rs 4,000 at 1% higher rate
of interest. If the interest in both the cases after 4 years is same the rate of interest in the
former case is:
(A) 4% (B) 5% (C) 1/4% (D) 8 1/3 %
Ans : A
5. A sum of money at simple interest rate amounts to Rs 4,025 in 3 years and to Rs 4,550
in years at the same
rate of interest. Find the sum and the rate of interest per annum.
(A) Rs 2,500, % (B) Rs 3,000, 5% (C) Rs 3,500, 5% (D) Rs 4,500, 4%
Ans : C
. On a certain map of India the actual distance of 1450 kms between two cities Delhi and
Kolkata is shown as
5 cms. What scale is used to draw the map?
(A) 1 : 15 10 (B) 1 : 20 10 (C) 1 : 25 10 (D) 1 : 29 10
Ans : D
7. The ratio between the third proportional of 12 and 30 and mean proportional of 9 and 25
is:
(A) 2 : 1 (B) 5 : 1 (C) 7 : 15 (D) 9 : 14
Ans : B
8. Rs 1,050 are divided among P, Q and R. The share of P is 2/5 of the combined share of Q
and R. P gets:
(A) Rs 320 (B) Rs 300 (C) Rs 200 (D) Rs 420
Ans : B
ma + nc
9. If a : b = c : d, then ________ is equal to:
mb + nd
(A) an : mb (B) m : n (C) a : b (D) dm : cn
Ans : C
10. How many even numbers of four-digits can be formed with digits 1, 2, 3, 4, 5,
(repetition of the digit is
allowed)?
(A) 48 (B) 180 (C) 129 (D) 540
Ans : A
11. There is a number lock with four rings. How many attempts at the maximum would
have to be made before
getting the right number?
(A) 104 (B) 255 (C) 104 - 1 (D) 25
Ans : C
12. There are four letters and four envelops addressed to different persons. In how many
ways can wrong choices
be made?
(A) 4 (B) 23 (C) 1 (D) 255
Ans : D
13. There are 10 points on a straight line AB and 8 points on another AC, none of them
being A. How many
triangles can be formed with these points as vertices?
(A) 720 (B) 40 (C) 81 (D) 80
Ans : B
14. A circular ground of circumference of 88m. A strip of land 3m wide inside and along the
circumference is to be
leveled. What is the expenditure if leveling cost is Rs. 7 per metre square.
Ans 650
15. There are 4 boys and 3 girls. What is the probability the boys and girls sit alternately?
Ans /35
1. Two trains are 2 kms apart. Speed of one train is 20m/s and the other train is running
at 30 m/s . Lengths of the
trains are 200 and 300m. In how much time do the trains cross each other?
Ans 50 seconds
17. A& B are two players. They need to select one number from 1 to 25. If both the players
select the same
numbers they will win the prize. What is the probability of not winning in a single trial?
Ans 24/25
18. Four different integers are in increasing AP such that one number is equal to sum of the
squares of the other
three numbers.
Ans -,0,,2
19. In a company there were 75 % skilled employees and the remaining unskilled. 80% of
the skilled and 20% of
the unskilled were permanent. If the temporary employees were 12 find the total
number of employees.
Ans 360
20. A person sells a horse at 12.5% loss. If he sells for 92.5 more, he will have a profit of
%. What is the CP?
Ans 500
21. One tap takes 15 min to fill, another tap takes 12 min to fill and the third tap can empty
in 20 min. In how much
time the tank would be full.
Ans 0 min
22. Two trains are separated by 200km. One leaves at :00 am from Delhi and reaches
Merrut at 10:00 am.
Another train leaves from Merrut at 8:00 am and reaches Delhi at 11:30 am. At what
time two trains meet each
other?
Ans 8:56 am
23. If the height of the triangle decreases by 40% and the breadth increases by 40%. Then
the effect on the area is
Ans 6% decrease
24. A grandfather has 5 sons and daughters and 8 grandchildren.. They have to be arranged
in arrow such that the
first 4 seats and last four seats are to be taken by grandchildren and the grandfather
would not sit adjacent to
any of the grandchildren.
Ans 4 * 8! * 5!
25. A farmer has a rectangular plot. He wants to do fencing along one of the side with the
help of the posts. Two
posts being on two corners. He brings 5 post less than what he has initially plan
because of which the distance
between two consecutive post became 8 m instead of m.. What is the length of the
side and no of post?
Ans 20 , 6
2. A circular ground of circumference of 88m. A strip of land 3m wide inside and along the
circumference is to be
leveled. What is the expenditure if leveling cost is Rs. 7 per metre square ?
Ans 650
27. Four horses are tethered at the four corners of a square of side 14cm such that two
horses along the same side
can just reach each other. They were able to graze the area in 11 days. How many days
will they take in order
to graze the left out area?
Ans 3
28. Six bells commence tolling together and toll at intervals 2,4,,8,10 and 12 seconds
respectively. In 30 minutes
how many times they toll together.
a) 4 b) 10 c) 15 d) 1
Ans: d)
29. The number of coins 1.5 cm in diameter and 0.2cm thick to be melted to form a right
circular cylinder of height
10 cm and diameter 4.5 cm is:
a) 380 b) 450 c) 472 d) 540
Ans: b)
30. The size of a wooden block is 5 10 20 cm3. How many whole such blocks you will
take to construct a solid
wooden cube of minimum size?
a) b) 8 c) 12 d) 1
Ans: b)
31. Find the odd man out - 1050, 510, 242, 10, 4, 1, 3
a) 510 b) 242 c) 10 d) 4
Ans: C)
32. A clock loses 10 minutes each hour. If the clock is set correctly at noon, what time is it
when it reads 3 PM?
Ans: 8 minutes
33. If 8 crows can steal 8 buttons in 8 minutes, how long will it take 1 crows to steal 1
buttons?
32 buttons
34. The amount of water in a tank doubles every minute. The tank is full in an hour. When
was the tank half full?
Ans: 59 minutes
35. A laborer can dig a hole 8 feet square and 8 feet deep in 8 days. How long will it take
him to dig a hole 4 feet
square and 4 feet deep?
Ans: day
3. A clock loses 10 minutes each hour. If the clock is set correctly at noon, what time is it
when it reads 3 PM?
Ans: 3:36 PM
37. If 2 miles of fence enclose a square plot of 10 acres, how large a square will 4 miles of
fence enclose?
Ans: 640 acres or square mile
38. A woman travels 1 mile south, then one mile west, and then mile north, and arrives at
her starting point.
Where is she?
Ans: North
$ection 3: CR%C RE$OG ( 2 questions , 30 minutes)
Directions-2: Answer the questions given below the passage or statement as
true, false or can't say
PASSAE : Copernicus is the intelligent. In the days of Copernicus the transport and
technology development was less & it took place weeks to communicate a message at that
time, wherein we can send it through satellite with in no time. Even with this fast
developments it has become difficult to understand each other.
Answer questions 1-4 based on passage .
1. People were not intelligent during Copernicus days
Ans False
2. Transport facilities are very much improved in noe a days
Ans Can't say
3. Even with the fast developments of the technology we can't live happily.
Ans Can't say
4. We can understand the people very much with the development of communication
Ans False
3 Critical Reasoning ( 2 questions , 30 minutes)
PASSAE: Senior managers warned the workers that because of the introductory of
Japanese industry in the car market. There is the threat to the workers. They also said that
there will be the reduction in the purchase of the sales of car in public. the interest rates of
the car will be increased with the loss in demand.
Answer questions 5-8 based on passage .
5. Japanese workers are taking over the jobs of Indian industry.
Ans False
.Managers said car interests will go down after seeing the raise in interest rates.
Ans True
7. Japanese investments are ceasing to end in the car industry.
Ans False
8. People are very interested to buy the cars.
Ans False
PASSAE: There should be copyright for all arts. The reels has came that all the arts has
come under one copy right society, they were use the money that come from the arts for
the developments . There may be a lot of money will come from the Tagore works. We have
to ask the benifiters from Tagore work to help for the development of his works.
Answer questions 35-39 based on passage .
9. Tagore works are came under this copy right rule.
Ans False
10. People are free to go to the public because of the copy right rule.
Ans Can't say

11. People gives to theater and collect the money for development.
Ans Can't say

12. We have ask the Tagore residents to help for the developments of art.
AnsCan't say

TCS -IV
1) If log 0.317=0.3332 and log 0.318=0.334 then find log 0.319 ?
Sol) log 0.317=0.3332 and log 0.318=0.334, then
log 0.319=log0.318+(log(0.318-0.317)) = 0.339
2) A box of 150 packets consists of 1kg packets and 2kg packets. Total weight of box is
24kg. How many 2kg packets are there ?
Sol) x= 2 kg Packs
y= 1 kg packs
x + y = 150 .... Eqn 1
2x + y = 24 .... Eqn 2
Solve the Simultaneous equation; x = 114
so, y = 3
ANS : Number of 2 kg Packs = 114.
3) My flight takes of at 2am from a place at 18N 10E and landed 10 Hrs later at a place with
coordinates 3N70W. What is the local time when my plane landed?
:00 am b) :40am c) 7:40 d) 7:00 e) 8:00
Sol) The destination place is 80 degree west to the starting place. Hence the time difference
between these two places is 5 hour 20 min. (=24hr80/30).
When the flight landed, the time at the starting place is 12 noon (2 AM + 10 hours).
Hence, the time at the destination place is 12 noon - 5:20 hours = : 40 AM
4) A plane moves from 9N40E to 9N40W. If the plane starts at 10 am and takes 8 hours
to reach the destination, find the local arrival time ?
Sol) Since it is moving from east to west longitide we need to add both
ie,40+40=80
multiply the ans by 4
=>804=320min
convert this min to hours ie, 5hrs 20min
It takes 8hrs totally . So 8-5hr 20 min=2hr 40min
So the ans is 10am+2hr 30 min
=>ans is 12:30 it will reach
5) The size of the bucket is N kb. The bucket fills at the rate of 0.1 kb per millisecond. A
programmer sends a program to receiver. There it waits for 10 milliseconds. And response
will be back to programmer in 20 milliseconds. How much time the program takes to get a
response back to the programmer, after it is sent? Please tell me the answer with
explanation. Very urgent.
Sol) see it doesnt matter that wat the time is being taken to fill the bucket.after reaching
program it waits there for 10ms and back to the programmer in 20 ms.then total time to
get the response is 20ms +10 ms=30ms.its so simple..
) A file is transferred from one location to another in `buckets. The size of the bucket is 10
kilobytes. Each bucket gets filled at the rate of 0.0001 kilobytes per millisecond. The
transmission time from sender to receiver is 10 milliseconds per bucket. After the receipt of
the bucket the receiver sends an acknowledgement that reaches sender in 100 milliseconds.
Assuming no error during transmission, write a formula to calculate the time taken in
seconds to successfully complete the transfer of a file of size N kilobytes.
(n/1000)(n/10)10+(n/100)..as i hv calculated.~~!not 100% sure
7) A fishermans day is rated as good if he catches 9 fishes, fair if 7 fishes and bad if 5
fishes. He catches 53 fishes in a week n had all good, fair n bad days in the week. So how
many good, fair n bad days did the fisher man had in the week
Ans:4 good, 1 fair n 2 bad days
Sol) Go to river catch fish
49=3
71=7
25=10
3+7+10=53.
take what is given 53
good days means - 9 fishes so 53/9=4(remainder=17) if u assume 5 then there is no
chance for bad days.
fair days means -- 7 fishes so remaining 17 - 17/7=1(remainder=10) if u assume 2 then
there is no chance for bad days.
bad days means ---5 fishes so remaining 10-10/5=2days.
Ans: 4 good, 1 fair, 2bad. ==== total 7 days.
x+y+z=7--- eq1
9x+7y+5z=53 ---eq2
multiply eq 1 by 9,
9x+9y+9z=35 -----eq3
from eq2 and eq3
2y+4z=10--eq4
since all x,y and z are integer i sud put a integer value of y such that z sud be integer in eq
4 ...and ther will be two value y=1 or 3 then z = 2 or 1 from eq 4
for first y=1,z=2 then from eq1 x= 4
so 94+17+25=53.. satisfied
now for second y=3 z=1 then from eq1 x=3
so 93+37+15=53 ..satisfied
so finally there are two solution of this question
(x,y,z)=(4,1,2) and (3,3,1).
Y catches 5 times more fishes than X. If total number of fishes caught by X and Y is 42,
then number of fishes caught by X?
Sol) Let no. of fish x catches=p
no. caught by y =r
r=5p.
r+p=42
then p=7,r=35
9) Three companies are working independently and receiving the savings 20%, 30%, 40%.
If the companies work combinely, what will be their net savings?
suppose total income is 100
so amount x is getting is 80
y is 70
z =0
total=210
but total money is 300
300-210=90
so they are getting 90 rs less
90 is 30% of 300 so they r getting 30% discount
10) The ratio of incomes of C and D is 3:4.the ratio of their expenditures is 4:5. Find the
ratio of their savings if the savings of C is one fourths of his income?
Sol) incomes:3:4
expenditures:4:5
3x-4y=1/4(3x)
12x-1y=3x
9x=1y
y=9x/1
(3x-4(9x/1))/((4x-5(9x/1)))
ans:12/19
11) If G(0) = -1 G(1)= 1 and G(N)=G(N-1) - G(N-2) then what is the value of G()?
ans: -1
bcoz g(2)=g(1)-g(0)=1+1=2
g(3)=1
g(4)=-1
g(5)=-2
g()=-1
12) If A can copy 50 pages in 10 hours and A and B together can copy 70 pages in 10
hours, how much time does B takes to copy 2 pages?
Sol) A can copy 50 pages in 10 hrs.
A can copy 5 pages in 1hr.(50/10)
now A & B can copy 70 pages in 10hrs.
thus, B can copy 90 pages in 10 hrs.[eqn. is (50+x)/2=70, where x-> no. of pages B can
copy in 10 hrs.]
so, B can copy 9 pages in 1hr.
therefore, to copy 2 pages B will need almost 3hrs.
since in 3hrs B can copy 27 pages.
13) whats the answer for that :
A, B and C are 8 bit nos. They are as follows:
A -> 1 1 0 0 0 1 0 1
B -> 0 0 1 1 0 0 1 1
C -> 0 0 1 1 1 0 1 0 ( - =minus, u=union)
Find ((A - C) u B) =?
To find A-C, We will find 2s compliment of C and them add it with A,
That will give us (A-C)
2s compliment of C=1s compliment of C+1
=11000101+1=11000110
A-C=11000101+11000110
=10001001
Now (A-C) U B is .OR. logic operation on (A-C) and B
10001001 .OR . 00110011
The answer is = 10111011,
Whose decimal equivalent is 187.
14) One circular array is given(means memory allocation tales place in circular fashion)
diamension(9X7) and sarting add. is 3000, What is the address of (2,3)....
Sol) its a 97 int array so it reqiure a 12 bytes for storing.bze integer value need 2 byes
of memory allocation. and starting add is 3000
so starting add of 23 will be 3012.
15) In a two-dimensional array, X (9, 7), with each element occupying 4 bytes of memory,
with the address of the first element X (1, 1) is 3000, find the address of X (8, 5).
Sol) initial x (1,1) = 3000 u hav to find from x(8,1)so u have x(1,1),x(1,2) . x(7,7) = so u
have totally 7 7 = 49 elementsu need to find for x(8,5) ? here we have 5 elements each
element have 4 bytes : (49 + 5 -1) 4 = 212 --( -1 is to deduct the 1 element ) 3000 +
212 = 3212
1) Which of the following is power of 3 a) 2345 b) 9875 c) 504 d) 9833
17) The size of a program is N. And the memory occupied by the program is given by M =
square root of 100N. If the size of the program is increased by 1% then how much memory
now occupied ?
Sol) M=sqrt(100N)
N is increased by 1%
therefore new value of N=N + (N/100)
=101N/100
M=sqrt(100 (101N/100) )
Hence, we get M=sqrt(101 N)
18) 1)SCOOTER --- AUTOMOBILE- A. PART OF
2.OXYGEN---- WATER --- B. A Type of
3.SHOP STAFF--- FITTERS-- C. NOT A TYPE OF
4. BUG -----REPTILE-- D. A SUPERSET OF
1)B 2)A 3)D 4)C
19) A bus started from bustand at 8.00a m and after 30 min staying at destination, it
returned back to the bustand. the destination is 27 miles from the bustand. the speed of the
bus 50 percent fast speed. at what time it returns to the bustand this is the step by step
solution:
a bus cover 27 mile with 18 mph in =27/18= 1 hour 30 min. and it wait at stand =30 min.
after this speed of return increase by 50% so 50%of 18 mph=9mph
Total speed of returnig=18+9=27
Then in return it take 27/27=1 hour
then total time in joureny=1+1:30+00:30 =3 hour
so it will come at 8+3 hour=11 a.m.
So Ans==11 a.m
20) In two dimensional array X(7,9) each element occupies 2 bytes of memory.If the
address of first element X(1,1)is 1258 then what will be the address of the element X(5,8) ?
Sol) Here, the address of first element x[1][1] is 1258 and also 2 byte of memory is given.
now, we have to solve the address of element x[5][8], therefore, 1258+ 582 = 1258+80
= 1338 so the answer is 1338.
21) The temperature at Mumbai is given by the function: -t2/+4t+12 where t is the
elapsed time since midnight. What is the percentage rise (or fall) in temperature between
5.00PM and 8.00PM?
22) Low temperature at the night in a city is 1/3 more than 1/2 high as higher temperature
in a day. Sum of the low temperature and highest temp. is 100 degrees. Then what is the
low temp?
Sol) Let highest temp be x
so low temp=1/3 of x of 1/2 of x plus x/2 i.e. x/+x/2
total temp=x+x/+x/2=100
therefore, x=0
Lowest temp is 40
23) In Madras, temperature at noon varies according to -t^2/2 + 8t + 3, where t is elapsed
time. Find how much temperature more or less in 4pm to 9pm. Ans. At 9pm 7.5 more
Sol) In equestion first put t=9,
we will get 34.5.........(1)
now put t=4,
we will get 27..........(2)
so ans=34.5-27
=7.5
24) A person had to multiply two numbers. Instead of multiplying by 35, he multiplied by 53
and the product went up by 540. What was the raised product?
a) 780 b) 1040 c) 1590 d) 1720
Sol) x53-x35=540=> x=30 therefore, 5330=1590 Ans
25) How many positive integer solutions does the equation 2x+3y = 100 have?
a) 50 b) 33 c) 1 d) 35
Sol) There is a simple way to answer this kind of Qs given 2x+3y=100, take l.c.m of `x
coeff and `y coeff i.e. l.c.m of 2,3 ==then divide 100 with , which turns out 1 hence
answer is 1short cut formula- constant / (l.cm of x coeff and y coeff)
2) The total expense of a boarding house are partly fixed and partly variable with the
number of boarders. The charge is Rs.70 per head when there are 25 boarders and Rs.0
when there are 50 boarders. Find the charge per head when there are 100 boarders.
a) 5 b) 55 c) 50 d) 45
Sol)
Let a = fixed cost and k = variable cost and n = number of boarders
total cost when 25 boarders c = 2570 = 1750 i.e. 1750 = a + 25k
total cost when 50 boarders c = 500 = 3000 i.e. 3000 = a + 50k
solving above 2 eqns, 3000-1750 = 25k i.e. 1250 = 25k i.e. k = 50
therefore, substituting this value of k in either of above 2 eqns we get
a = 500 (a = 3000-5050 = 500 or a = 1750 - 2550 = 500)
so total cost when 100 boarders = c = a + 100k = 500 + 10050 = 5500
so cost per head = 5500/100 = 55
27) Amal bought 5 pens, 7 pencils and 4 erasers. Rajan bought pens, 8 erasers and 14
pencils for an amount which was half more than what Amal had paid. What % of the total
amount paid by Amal was paid for pens?
a) 37.5% b) 2.5% c) 50% d) None of these
Sol)
Let, 5 pens + 7 pencils + 4 erasers = x rupees
so 10 pens + 14 pencils + 8 erasers = 2x rupees
also mentioned, pens + 14 pencils + 8 erarsers = 1.5x rupees
so (10-) = 4 pens = (2-1.5)x rupees
so 4 pens = 0.5x rupees => 8 pens = x rupees
so 5 pens = 5x/8 rupees = 5/8 of total (note x rupees is total amt paid byamal)
i.e 5/8 = 500/8% = 2.5% is the answer
28) I lost Rs.8 in two races. My second race loss is Rs. more than the first race. My friend
lost Rs.4 more than me in the second race. What is the amount lost by my friend in the
second race?
Sol)
x + x+ = rs 8
2x + = 8
2x = 8-
2x = 2
x=31
x is the amt lost in I race
x+ = 31+=37 is lost in second race
then my friend lost 37 + 4 = 41 Rs
29) Ten boxes are there. Each ball weighs 100 gms. One ball is weighing 90 gms. i) If there
are 3 balls (n=3) in each box, how many times will it take to find 90 gms ball? ii) Same
question with n=10 iii) Same question with n=9
to me the chances are
when n=3
(i) nC1= 3C1 =3 for 10 boxes .. 103=30
(ii) 10C1=10 for 10 boxes ..1010=100
(iii)9C1=9 for 10 boxes ...109=90
30) (1-1/) (1-1/7).. (1- (1/ (n+4))) (1-(1/ (n+5))) = ?
leaving the first numerater and last denominater, all the numerater and denominater will
cancelled out one another. Ans. 5/(n+5)
31) A face of the clock is divided into three parts. First part hours total is equal to the sum
of the second and third part. What is the total of hours in the bigger part?
Sol) the clock normally has 12 hr
three parts x,y,z
x+y+z=12
x=y+z
2x=12
x=
so the largest part is hrs
32) With 4/5 full tank vehicle travels 12 miles, with 1/3 full tank how much distance travels
Sol) 4/5 full tank= 12 mile
1 full tank= 12/(4/5)
1/3 full tank= 12/(4/5)(1/3)= 5 miles
33) wind blows 10 miles in 330min.for 80 miles how much time required
Sol) 10 miles= 330 min
1 mile = 330/10
80 miles=(33080)/10=15 min.
34) A person was fined for exceeding the speed limit by 10mph.another person was also
fined for exceeding the same speed limit by twice the same if the second person was
travelling at a speed of 35 mph. find the speed limit
Sol) (x+10)=(x+35)/2
solving the eqn we get x=15
35) A sales person multiplied a number and get the answer is 3 instead of that number
divided by 3. what is the answer he actually has to get.
Sol) Assume 1
1 3 = 3
11/3=1/3
so he has to got 1/3
this is the exact answer
3) A person who decided to go weekend trip should not exceed 8 hours driving in a day
average speed of forward journey is 40 mph due to traffic in Sundays the return journey
average speed is 30 mph. How far he can select a picnic spot.
37) Low temperature at the night in a city is 1/3 more than 1/2 hinge as higher
temperature in a day. Sum of the low temp and high temp is 100 c. then what is the low
temp.
ans is 40 c.
Sol) let x be the highest temp. then,
x+x/2+x/=100.
therefore, x=0 which is the highest temp
and 100-x=40 which is the lowest temp.
38) car is filled with four and half gallons of oil for full round trip. Fuel is taken 1/4 gallons
more times in going than coming. What is the fuel consumed in coming up.
Sol) let feul consumed in coming up is x. thus equation is: x+1.25x=4.5ans:2gallons
39) A work is done by the people in 24 min. One of them can do this work alone in 40 min.
How much time required to do the same work for the second person
Sol) Two people work together in 24 mins.
So, their one day work is
(1/A)+(1+B)=(1/24)
One man can complete the work in 40mins
one mans one day work (1/B)= (1/40)
Now,
(1/A)=(1/24)-(1/40)
(1/A)=(1/0)
So, A can complete the work in 0 mins.
40) In a company 30% are supervisors and 40% employees are male if 0% of supervisors
are male. What is the probability? That a randomly chosen employee is a male or female?
Sol) 40% employees are male if 0% of supervisors are male so for 100% is 2.4%so the
probability is 0.24
41) In 80 coins one coin is counterfeit what is minimum number of weighing to find out
counterfeit coin
Sol) the minimum number of wieghtings needed is just 5.as shown below
(1) 80->30-30
(2) 15-15
(3) 7-7
(4) 3-3
(5) 1-1
42) 2 oranges, 3 bananas and 4 apples cost Rs.15. 3 oranges, 2 bananas, and 1 apple costs
Rs 10. What is the cost of 3 oranges, 3 bananas and 3 apples?
2x+3y+4z=15
3x+2y+z=10 adding
5x+5y+5z=25
x+y+z=5 that is for 1 orange, 1 bannana and 1 apple requires 5Rs.
so for 3 orange, 3 bannana and 3 apple requires 15Rs.
i.e. 3x+3y+3z=15
43) In 88 chess board what is the total number of squares refers
Sol) odele discovered that there are 204 squares on the board We found that you would add
the different squares - 1 + 4 + 9 + 1+ 25 + 3 + 49 + 4.
Also in 33 tic tac toe board what is the total no of squares
Ans 14 ie 9+4(bigger ones)+1 (biggest one)
If you ger 100100 board just use the formula
the formula for the sum of the first n perfect squares is
n x (n + 1) x (2n + 1)
______________________

if in this formula if you put n=8 you get your answer 204
44) One fast typist type some matter in 2hr and another slow typist type the same matter
in 3hr. If both do combinely in how much time they will finish.
Sol) Faster one can do 1/2 of work in one hourslower one can do 1/3 of work in one
hourboth they do (1/2+1/3=5/) th work in one hour.so work will b finished in /5=1.2
hour i e 1 hour 12 min.
45) If Rs20/- is available to pay for typing a research report & typist A produces 42 pages
and typist B produces 28 pages. How much should typist A receive?
Here is the answer Find of 42 % of 20 rs with respect to 70 (i.e 28 + 42) ==> (42 20
)/70 ==> 12 Rs
4) An officer kept files on his table at various times in the order 1,2,3,4,5,. Typist can
take file from top whenever she has time and type it.What order she cannt type.?
47) In some game 139 members have participated every time one fellow will get bye what
is the number of matches to choose the champion to be held?
the answer is 138 matches
Sol) since one player gets a bye in each round,he will reach the finals of the tournament
without playing a match. http://www.vyoms.com
therefore 137 matches should be played to detemine the second finalist from the remaining
138 players(excluding the 1st player)
therefore to determine the winner 138 matches shd be played.
48) One rectangular plate with length 8inches, breadth 11 inches and 2 inches thickness is
there. What is the length of the circular rod with diameter 8 inches and equal to volume of
rectangular plate?
Sol) Vol. of rect. plate= 8112=17
area of rod=(22/7)(8/2)(8/2)=(352/7)
vol. of rod=arealength=vol. of plate
so length of rod= vol of plate/area=17/(352/7)=3.5
49) One tank will fill in minutes at the rate of 3cu ft /min, length of tank is 4 ft and the
width is 1/2 of length, what is the depth of the tank?
3 ft 7.5 inches
50) A man has to get air-mail. He starts to go to airport on his motorbike. Plane comes
early and the mail is sent by a horse-cart. The man meets the cart in the middle after half
an hour. He takes the mail and returns back, by doing so, he saves twenty minutes. How
early did the plane arrive?
ans:10min:::assume he started at 1:00,so at 1:30 he met cart. He returned home at
2:00.so it took him 1 hour for the total jorney.by doing this he saved 20 min.so the actual
time if the plane is not late is 1 hour and 20 min.so the actual time of plane is at 1:40.The
cart travelled a time of 10 min before it met him.so the plane is 10 min early.
51) Ram singh goes to his office in the city every day from his suburban house. His driver
Mangaram drops him at the railway station in the morning and picks him up in the evening.
Every evening Ram singh reaches the station at 5 oclock. Mangaram also reaches at the
same time. One day Ram singh started early from his office and came to the station at 4
oclock. Not wanting to wait for the car he starts walking home. Mangaram starts at normal
time, picks him up on the way and takes him back house, half an hour early. How much
time did Ram singh walked?
52) 2 trees are there. One grows at 3/5 of the other. In 4 years total growth of the trees is
8 ft. what growth will smaller tree have in 2 years.
Sol) THE BIG TREE GROWS 8FT IN 4 YEARS=>THE BIG TREE GROWS 4FT IN 2
YEARS.WHEN WE DIVIDE 4FT/5=.83=>2.4
ans: 1.5 mt 4 (x+(3/5)x)=88x/5=2x=5/4 after 2 years x=(3/5)(5/4)2 =1.5
53) There is a six digit code. Its first two digits, multiplied by 3 gives all ones. And the next
two digits multiplied by give all twos. Remaining two digits multiplied by 9 gives all threes.
Then what is the code?
sol) Assume the digit xx xx xx (six digits)
First Two digit xx 3=111
xx=111/3=37
( first two digits of 1 is not divisible by 3 so we can use 111)
Second Two digit xx=222
xx=222/=37
( first two digits of 2 is not divisible by so we can use 222)
Thrid Two digit xx9=333
xx=333/9=37
( first two digits of 3 is not divisible by 9 so we can use 333)
54) There are 4 balls and 4 boxes of colours yellow, pink, red and green. Red ball is in a box
whose colour is same as that of the ball in a yellow box. Red box has green ball. In which
box you find the yellow ball?
ans is green.
Sol) Yellow box can have either of pink/yellow balls.
if we put a yellow ball in "yellow box then it wud imply that "yellow is also the colour of
the box which has the red ball(becoz acordin 2 d question,d box of the red ball n the ball in
the yellow box have same colour)
thus this possibility is ruled out.
therefore the ball in yellow box must be pink,hence the colour of box containin red ball is
also pink..
=>the box colour left out is "green,,,which is alloted to the only box left,,,the one which
has yellow ball..
55) A bag contains 20 yellow balls, 10 green balls, 5 white balls, 8 black balls, and 1 red
ball. How many minimum balls one should pick out so that to make sure the he gets at least
2 balls of same color.
Ans:he should pick ball totally.
Sol) Suppose he picks 5 balls of all different colours then when he picks up the sixth one, it
must match any on of the previously drawn ball colour. thus he must pick balls
5) What is the number of zeros at the end of the product of the numbers from 1 to 100
Sol) For every 5 in unit palce one zero is added Ch eta naS
so between 1 to 100 there are 10 nos like 5,15,25,..,95 which has 5 in unit place.
Similarly for every no divisible by 10 one zero is added in the answer so between 1 to 100
11 zeros are added
for 25,50,75 3 extra zeros are added so total no of zeros are 10+11+3=24
57) 10 Digit number has its first digit equals to the numbers of 1s, second digit equals to
the numbers of 2s, 3rd digit equals to the numbers of 3s .4th equals number of 4s..till 9th
digit equals to the numbers of 9s and 10th digit equals to the number of 0s. what is the
number?.(marks)
ans:210001000
2-shows that two 1s in the ans
1-shows that one 2 in ans
0-shows no 3 in the ans
0-shows no 4 in the ans
0-shows no 5 in the ans
1-shows one in the ans
0-shows no 7 in the ans
0-shows no 8 in the ans
0-shows no 9 in the ans
-shows six 0s in the ans
58) There are two numbers in the ratio 8:9. if the smaller of the two numbers is increased
by 12 and the larger number is reduced by 19 thee the ratio of the two numbers is 5:9. Find
the larger number?
sol) 8x:9x initialy
8x+ 12 : 9x - 19 = 5x:9x
8x+12 = 5x
-> x = 4
9x = 3 not sure about the answer ..
59) There are three different boxes A, B and C. Difference between weights of A and B is 3
kgs. And between B and C is 5 kgs. Then what is the maximum sum of the differences of all
possible combinations when two boxes are taken each time
A-B = 3
B-c = 5
a-c = 8
so sum of diff = 8+3+5 = 1 kgs
0) A and B are shooters and having their exam. A and B fall short of 10 and 2 shots
respectively to the qualifying mark. If each of them fired atleast one shot and even by
adding their total score together, they fall short of the qualifying mark, what is the
qualifying mark?
ans is 11
coz each had atleast 1 shot done so 10 + 1 = 11
n 9 + 2 = 11
so d ans is 11
1) A, B, C, and D tells the following times by looking at their watches. A tells it is 3 to 12.
B tells it is 3 past 12. C tells it is 12:2. D tells it is half a dozen too soon to 12. No two
watches show the same time. The difference between the watches is 2,3,4,5 respectively.
Whose watch shows maximum time?
sol) A shows 11:57, B shows 12:03, C shows 12:02, and D shows 11:0 therefore, max
time is for B
2) Falling height is proportional to square of the time. One object falls 4cm in 2sec than
in sec from how much height the object will fall.
Sol) The falling height is proportional to the squere of the time.
Now, the falling height is 4cm at 2sec
so, the proportional constant is=4/(22)=1;
so, at sec the object fall maximum (1)cm=57cm;
Now, the object may be situated at any where.
if it is>57 only that time the object falling 57cm within sec .Otherwise if it is
situated<57 then it fall only that height at sec.
3) Gavaskar average in first 50 innings was 50. After the 51st innings his average was 51
how many runs he made in the 51st innings
Ans) first 50 ings.- run= 5050=2500
51st ings.- avg 51. so total run =5151=201.
so run scored in that ings=201-2500=101 runs.
4) Anand finishes a work in 7 days, Bittu finishes the same job in 8 days and Chandu in
days. They take turns to finish the work. Anand on the first day, Bittu on the second and
Chandu on the third day and then Anand again and so on. On which day will the work get
over?
a) 3rd b) th c) 9th d) 7th
Ans is d) 7th day
Sol) In d 1st day Anand does 1/7th of total work
similarly,
Bithu does 1/8th work in d 2nd day
hence at d end of 3 days, work done = 1/7+1/8+1/=73/18
remaining work = (18-73)/18 = 95/18
again after days of work, remaining work is = (95-73)/18 = 22/18 and hence Anand
completes the work on 7th day.(hope u understood.)
5) A man, a women and a child can do a piece of work in days,man can do it in 14 days,
women can do it 1 days, and in how many days child can do the same work?
The child does it in 24 days
) A: 1 1 0 1 1 0 1 1
B: 0 1 1 1 1 0 1 0
C: 0 1 1 0 1 1 0 1
Find ( (A-B) u C )==?
Hint : 109
A-B is {A} - {A n B}
A: 1 1 0 1 1 0 1 1
B: 0 1 1 1 1 0 1 0
by binary sub. a-b = 01100001 (1-0=1, 1-1=0,0-0=0, n for the 1st 3 digits 110-011=011)
now (a-b)uc= 01100001
or 01101101
gives 1101101. convert to decimal equals 10
CRITICAL REASONING
this section is very important there are 12 question and u have to solve it in 30 minutes..
for critical reasoning ..go thru the barrons 5 model test paper..just mug up all the answer
but beware ...they can change the character names so be careful......
after clearing my written they called us for interview on 12th march
TECHNICAL INTERVIEW
after waiting for more than seven hours they called me for the interview .look just be
confident and have eye contact with them ...actually in the starting i was very much
confident and answered all his question but after some time dont knw wat he asked to me
and and my mind was just blocked...dont knw y..even this is not the end .when i
finished my tech interview and came out of the hall ...i forgot my file there... ...well
after few minutes i called for the H.R.
H.R.
same things .introduce urself, strengths weakness,future plan,where do u see urself after 5
years...i told them that i am interested in embedded ..they asked if we send u u.s. will u
go?...and some more h.r. question..
i advice u guys dont even try to make them fool .if u dont knw any answer ..just tell them
`u dont knw
M.R.

También podría gustarte